shape
shape
  • Home
  • Blog
  • UPSC Prelims Answer Key 2024 With Explanation (GS Paper-1)
UPSC Prelims Answer Key 2024 With Explanation (GS Paper-1)

UPSC Prelims Answer Key 2024 With Explanation (GS Paper-1)

UPSC Prelims 2024 was held on 16th June, 2024. The Preliminary Examination is of objective type; it consists of two papers: Paper-I (General Studies) and Paper-II (CSAT). Paper-II is a qualifying paper with minimum qualifying marks of 33% and would not be considered for merit; only score in Paper-I would be considered for selecting candidates for the Main Examination.

UPSC Prelims, 2024 is considered as an easy paper when compared to the previous five years. The paper came as a surprise to the aspirants and the teaching community; although the paper was easy at the periphery, there is much scope for negative marking as mistakes can be easily committed given the nature of language used and combination of options given.

Geography

There are 17 Questions from Geography, of which 

  • 9 Questions on Physical Geography, which are directly picked from NCERT and G.C.Leong 
  • 1 Question on Economic Geography 
  • 5 Questions on World Geography, of which two are map-based 
  • 2 Questions from Indian Geography, which are map-based 

The level of the questions is moderate.

Most of the questions are aimed at testing the basic understanding of the subject and conceptual clarity.

Environment

There are 17 Questions from Environment, of which 

  • 5 Questions are related to Climate Change
  • 5 Questions are related to Environment and Ecology 
  • 7 Questions are related to Bio-Diversity 

The examiner covered areas such as the Fundamentals of Climate Change, Global Climate issues, Climate Change Initiatives, Alternative and Sustainable Fuels, the Basics of the Environment, the basics of Biodiversity, Biodiversity in India, and Global Biodiversity and Environmental Organizations. 

The level of the questions is moderate.

Questions aim to test the basic and applied section of Environment Bio-diversity, Climate-change and Environment.

Science & Technology

There are 10 Questions from Science and Technology.

The examiner tested students’ understanding of current technological developments in the fields of Energy, Electronics, Defence, Medical, and Alternative Fuels. 

The level of questions is moderate to tough.

Questions aimed to test the candidate’s knowledge of the Application of Science and Technology in day-to-day life. 

Economy

There are 13 Questions from the Economy, of which 

  • 7 Question are from Money Market and Capital Market covering the basic and applied aspects 
  • 3 Questions are based on the Basics of Economy
  • 2 Questions are on Banking & RBI 
  • 1 Question on International Institutions. 

Overall the level of Questions is easy to moderate. 

The examiner’s focus is to test the understanding of basic concepts, and their application.

History

There are 10 Questions from History, of which 

  • 5 Questions are asked from Ancient India, of which 3 Questions are on Ancient Indian literature and a Question each from Ancient Indian Religions and Archeological Sites.
  • Only 1 Question from Medieval India (from Vijayanagar)
  • 3 Questions from Modern India of which 2 Questions on Constitutional Development and a Question on Land Tenures during British era (Economic History) 
  • 1 Question on Post Independence 

Overall the level of Questions can be considered as moderate to tough. 

The questions framed by the examiner are in such a way as to test a Comprehensive understanding of history, comprising Political, Religious, Intellectual/Institutional, Cultural, Economic and Social aspects. 

Polity

There are 17 Questions are asked in Polity, of which 

  • 6 Questions on Parliament and Parliamentary Procedures.
  • 2 Questions on Statutory Bodies. 
  • 2 Questions on Ministries and Departments 
  • 2 Questions on Fundamental Rights and Writs
  • 1 Question on Constitutional Amendment.
  • 1 Question on Special Provisions for Scheduled Tribes
  • 1 Question on Parts of the Constitution

Overall, the level of Questions is Easy to Moderate. 

Questions are framed to test the Candidate’s complete understanding of parts and provisions of the Constitution, Parliament and Parliamentary Procedures.

Current Affairs

There are 16 Questions from Current Affairs, of which 

  • 5 Questions are from Government Schemes and Initiatives
  • 4 Questions on International Issues 
  • 2 Questions on Art and Culture 
  • 1 Question each on  Economy, Environment, Bilateral issues, Human Geography, Books and authors. 

The level of the questions is easy to moderate. 

The examiner wanted to check the candidate’s awareness of issues of national and international importance and current affairs related to polity, environment, history, economy, geography, science, and technology. 

Attempting 90 questions can be considered as a Good attempt.

100 is considered a good score for GS Paper I, and cut-off marks for clearing Prelims would be around  95(+/- 2 )

Download the UPSC Prelims Answer Key PDF for all sets of General Studies Paper 1 prepared by Kranthi Pavel Irigi Sir (Director – Max IAS) and his team for the reference of the candidates who appeared for UPSC Prelims 2024 as well as aspirants appearing for upcoming competitive exams.

GS Paper 1 SETUPSC Answer Key PDF
GS Paper 1, Set APrelims GS 1 Answer Key 2024 PDF
GS Paper 1, Set B
GS Paper 1, Set C
GS Paper 1, Set D
GS Paper 1UPSC Answer Key with Explanation PDF
GS Paper 1, Set AClick here to Download
GS Paper 1, Set BClick here to Download
GS Paper 1, Set CClick here to Download
GS Paper 1, Set DClick here to Download
UPSC Prelims Answer Key 2024 With Explanation (GS Paper-1)

1. Consider the following statements:

Statement-I: The atmosphere is heated more by incoming solar radiation than by terrestrial radiation.

Statement-ll: Carbon dioxide and other greenhouse gases in the atmosphere are good absorbers of long-wave radiation.

Which one of the following is correct in respect of the above statements?

(a) Both Statement-I and Statement-II are correct and Statement-II explains Statement-I

(b) Both Statement-I and Statement-II are correct, but Statement-II does not explain Statement-I

(c) Statement-I is correct, but Statement-II is incorrect

(d) Statement-I is incorrect, but Statement-II is correct

1. Ans: d

Explanation:

The insolation (incoming solar radiation) received by the earth is in short waveforms and heats up its surface. The earth after being heated itself becomes a radiating body and it radiates energy to the atmosphere in the long waveform. This energy heats up the atmosphere from below. This process is known as terrestrial radiation.

Hence, statement 1 is incorrect.

The long wave radiation is absorbed by the atmospheric gases particularly by carbon dioxide and the other greenhouse gases. Thus, the atmosphere is indirectly heated by the Earth’s radiation.

Hence, statement 2 is correct.

2. Consider the following statements:

Statement-I: Thickness of the troposphere at the equator is much greater as compared to the poles.

Statement II: At the equator, heat is transported to great heights by strong convectional currents.

Which one of the following is correct in respect of the above statements?

(a) Both Statement-I and Statement-II are correct and Statement-II explains Statement-I

(b) Both Statement-I and Statement-II are correct, but Statement-II does not explain Statement-I

(c) Statement-I is correct, but Statement-II is incorrect

(d} Statement-I is incorrect, but Statement-II is correct.

2. Ans: a

Explanation:

The troposphere is the lowermost layer of the atmosphere. Its average height is 13 km and extends roughly to a height of 8 km near the poles and about 18 km at the equator. The thickness of the troposphere is greatest at the equator because heat is transported to great heights by strong convectional currents.

This layer contains dust particles and water vapour. All changes in climate and weather take place in this layer.

The temperature in this layer decreases at the rate of 1° C for every 165m of height. This is the most important layer for all biological activity.

Both statements 1 & 2 are correct, and statement 2 correctly explains statement 1.

3. Consider the following:

1. Pyroclastic debris

2. Ash and dust

3. Nitrogen compounds

4. Sulphur compounds

How many of the above are products of volcanic eruptions?

(a) Only one

(b) Only two

(c) Only three

(d) All four

3. Ans: d

Explanation:

The material that reaches the ground includes lava flows, pyroclastic debris, volcanic bombs, ash and dust and gases such as nitrogen compounds, sulphur compounds and minor amounts of chlorine, hydrogen and argon. 

Hence, option d is correct. 

4. Which of the following is/are correct inference/inferences from isothermal maps in the month of January?

1. The isotherms deviate to the north over the ocean and to the south over the continent.

2. The presence of cold ocean currents, Gulf Stream and North Atlantic Drift makes the North Atlantic Ocean colder and the isotherms bend towards the north.

Select the answer using the code given below:

(a) 1 only

(b) 2 only

(c) Both 1 and 2

(d) Neither 1 nor 2

4. Ans: a

Explanation:

The temperature distribution is generally shown on the map with the help of isotherms. The Isotherms are lines joining places having equal temperatures. In general, the effect of the latitude on temperature is well pronounced on the map, as the isotherms are generally parallel to the latitude. The deviation from this general trend is more pronounced in January than in July, especially in the northern hemisphere. In the northern hemisphere the land surface area is much larger than in the southern hemisphere. Hence, the effects of land mass and the ocean currents are well-pronounced. In January the isotherms deviate to the north over the ocean and to the south over the continent. This can be seen in the North Atlantic Ocean. The presence of warm ocean currents, Gulf Stream and North Atlantic drift, make the Northern Atlantic Ocean warmer and the isotherms bend towards the north.

Statement 1 is correct and Statement 2 is incorrect.

5. Which of the following countries are well known as the two largest cocoa producers in the world?

(a) Algeria and Morocco

(b) Botswana and Namibia

(c) Côte d’Ivoire and Ghana

(d) Madagascar and Mozambique

5. Ans: c

Explanation:

Côte d’Ivoire (the Ivory Coast) is the largest producer of cocoa in the world, producing over 2 million tons a year. Ghana ranks second in the world. 

Hence, option c is correct. 

6. With reference to the Himalayan rivers joining the Ganga downstream of Prayagraj from West to East, which one of the following sequences is correct?

(a) Ghaghara – Gomati – Gandak – Kosi

(b) Gomati – Ghaghara – Gandak – Kosi

(c) Ghaghara – Gomati – Kosi – Gandak

(d) Gomati – Ghaghara – Kosi – Gandak

6. Ans: b

Explanation:

UPSC Prelims Answer Key 2024 With Explanation 1
Image Source: NCERT

Gomti: The Gomti river originates near Manikot in the Pillibhit district of Uttar Pradesh. From its origin to its confluence with Ganga, the river flows entirely in the State of Uttar Pradesh. Lucknow the capital city of Uttar Pradesh is situated on the banks of Gomti and it joins the Ganga in Audihar in the Jaunpur district of Uttar Pradesh.

Ghaghara: The Ghaghara river known variously as the Sarju or the Dehwa contains the combined waters of the Chauka or Sarda and the Kauriala which unite near Bahramghat in the Baranki district in Uttar Pradesh. 

The Sarda River is the important tributary of the Ghaghara, which forms the boundary between India and Nepal for some distance. The total length of the Ghaghara before its confluence with the Ganga River at Doriganj downstream of Chapra town in Bihar is 1,080 km. 

Gandak: The Gandak river (Sadanira, Saligrami in Nepal, or Narayani in the plains), known as the Kali or Krishna Gandak in the upper reaches rises at the altitude of 7620 m in Tibet near the Nepal border.   It cuts through the Mahabharat Range and covering a distance of 425 km joins the Ganga near Patna.

Kosi: The Kosi (Kausika) is the largest of the tributaries of the Ganga is formed by the confluence of three streams namely the Sun Kosi, the Arun Kosi and the Tamur Kosi, all taking their origin in the Himalayan region of Nepal and Tibet. 

The Kosi is notorious for its frequent and disastrous floods and shifting of courses. It is also called as ‘the sorrow of Bihar’ and meets the Ganga 32 km west of Manihari.

Hence, option b is correct. 

7. Consider the following statements:

Statement I: Rainfall is one of the reasons for the weathering of rocks.

Statement-II: Rainwater contains carbon dioxide in solution.

Statement-III: Rainwater contains atmospheric oxygen. 

Which one of the following is correct in respect of the above statements?

(a) Both Statement-II and Statement-III are correct and both of them explain Statement-I

(b) Both Statement-II and Statement-III are correct, but only one of them explains Statement-I

(c) Only one of Statement II and III is correct and that explains Statement-I 

(d) Neither Statement-II nor Statement-III is correct

7. Ans: a

Explanation:

Chemical weathering

Chemical weathering is the basic process by which denudation proceeds. It is the extremely slow and gradual decomposition of rocks due to exposure to air and water. Soil absorbs rainwater and keeps the underlying rocks in contact with this moisture. The rainwater absorbs organic acids from the soil and thus becomes a stronger weathering agent.

Statement 1 is correct.

There are three major chemical weathering processes.

  1. Solution: many minerals are dissolved by water, especially when, as with rainwater, it contains enough carbon dioxide to make it a weak acid. The solution is the most potent weathering process in limestone regions because the rainwater attacks and dissolves the calcium carbonate from which the rock is chiefly formed.

Statement 2 is correct.

  1. Oxidation: Oxidation is the reaction of oxygen in air or water with minerals in rocks. For example, most rocks contain a certain amount of iron, which, when it comes in contact with air, is changed to iron oxide, a familiar brownish crust or rust.

Statement 3 is correct.

  1. Decomposition by organic acids: with the soil, which covers most rocks are bacteria that thrive on decaying plants or animal materials. These bacteria produce acids which, when dissolved in water, help to speed up the weathering of the underlying rocks.

Both Statement II and Statement III are correct and both of them explain Statement I

8. Consider the following countries:

1. Finland

2. Germany

3. Norway

4. Russia

How many of the above countries have a border with the North Sea?

(a) Only one

(b) Only two

(c) Only three

(d) All four

8. Ans: b

Explanation: 

The North Sea, a shallow, northeastern arm of the Atlantic Ocean, is located between the British Isles and the mainland of northwestern Europe and covers an area of 2,20,000 square miles (5,70,000 square km). 

The sea is bordered by the island of Great Britain to the southwest and west, the Orkney and Shetland islands to the northwest, Norway to the northeast, Denmark to the east, Germany and the Netherlands to the southeast, and Belgium and France to the south. It is connected to the Atlantic by the Strait of Dover and the English Channel and opens directly onto the ocean between the Orkney and Shetland Islands and between the Shetland Islands and Norway. The Skagerrak, an eastward extension of the North Sea between Norway and Denmark, connects the North and Baltic seas via the Kattegat and the Danish straits.

Hence, option b is correct. 

9. Consider the following information:

WaterfallRegionRiver
1.DhuandharMalwaNarmada
2.HundruChota NagpurSubarnarekha
3.GersoppaWestern GhatsNetravati

In how many of the above rows is the given information correctly matched?

(a) Only one

(b) Only two

(c) All three

(d) None

9. Ans: a

Explanation: 

WaterfallRegionRiver
1.DhuandharVindhyanchal Baghelkhand regionNarmada
2.HundruChota NagpurSubarnarekha
3.GersoppaWestern GhatsShravati

Narmada

Narmada is the largest west-flowing river of the peninsula India. It rises from a Kund near Amarkantak, in the Anuppur district of Madhya Pradesh, at an elevation of about 1057 m in the Maikala range. The river flows through Madhya Pradesh, Maharashtra and Gujarat between Vindhya and Satpura hill ranges before falling into the Gulf of Cambay in the Arabian Sea about 10 km north of Bharuch.

Flowing in a rift valley between the Satpura in the south and the Vindhyan range in the north, Narmada forms a picturesque gorge in marble rocks and the Dhuandhar waterfall near Jabalpur. 

Vindhyanchal Baghelkhand region – The region is a hill-valley complex covering Annupur, Shahdol, Dindori, Mandla, Balaghat, Jabalpur, Narsimhapur and Chhindwara districts of Madhya Pradesh falling in Narmada basin.

The Subarnarekha is one of the longest east-flowing inter-state rivers. It originates near Nagri village in Ranchi district of Jharkhand at an elevation of 600 m. The total length of the river is about 395 km. The principal tributaries of the river are Kanchi, Kharkai and Karkari. The Subernarekha basin extends over the States of Jharkhand, Odisha and a comparatively smaller part of West Bengal. 

The Hundru Falls in Ranchi is created on the course of the Subarnarekha River, where it falls from a height of 320 feet creating the highest waterfalls in the state of Jharkhand.

The River Sharavathi – The streams of the Western Ghats flow swiftly down the steep slope and some of them make waterfalls. The Jog or Gersoppa Falls (289m) made by the Sharavati river is the most famous waterfall in India.

The River Sharavathi flows across Uttara Kannada and the Shimoga District of Karnataka. Originating at Ambutirthha (Tirthahalli), it flows for nearly 128 km before joining the Arabian Sea at Karki, Honnavar.

The plain regions of the catchment are dominated by lakes whereas the Ghats are dominated by streams.

The variations in the terrain have led to the formation of various waterfalls such as the Jog Falls/ Gersoppa, Apsarakonda, Mavinagundi falls, and Dabbe fall.

Only 2nd row is correctly matched.

Hence, option a is correct.

10. Consider the following information:

RegionName of the mountain rangeType of mountain
1.Central AsiaVosgesFold mountain
2.EuropeAlpsBlock mountain
3.North AmericaAppalachiansFold mountain
4.South AmericaAndesFold mountain

In how many of the above rows is the given information correctly matched?

(a) Only one

(b) Only two

(c) Only three

(d) All four

10. Ans: b

Explanation:

RegionName of the Mountain RangeType of Mountain
1.EuropeVosgesBlock Mountain
2.EuropeAlpsFold Mountain
3.North AmericaAppalachiansFold Mountain
4.South AmericaAndesFold Mountain

Fold Mountains: They are caused by large-scale earth movements when stresses are set up in the earth’s crust. When stresses are initiated, the rocks are subjected to compressive forces that produce wrinkling or folding along the lines of weakness.

The great fold mountains of the world are the Himalayas, Rockies, Appalachians, Andes and Alps.

Block Mountains: When the earth’s crust bends folding occurs, but when it cracks, faulting takes place. Faulting may be caused by tension or compression, forces that lengthen or shorten the earth’s crust, causing a section of it to subside or to rise above the surrounding level. 

Examples of block mountains include the Hunsruck Mountains, the Vosges and the Black Forest of the Rhineland.

  • Vosges, massif extending west of the Rhine River Valley in the Haut-Rhin, Bas-Rhin, and Vosges départements of eastern France (Europe).
  • Alps: The Alps form part of France, Italy, Switzerland, Germany, Austria, Slovenia, Croatia, Bosnia and Herzegovina, Montenegro, Serbia, and Albania (Europe).
  • Appalachian Mountains:  They are the great highland system of North America, the eastern counterpart of the Rocky Mountains. Extending for almost 2,000 miles (3,200 km) from the Canadian province of Newfoundland and Labrador to central Alabama in the United States, the Appalachian Mountains form a natural barrier between the eastern Coastal Plain and the vast Interior Lowlands of North America.
  • Andes Mountains Range: The Andes Mountains are the longest continental mountain range in the world. It forms a continuous highland along the western edge of South America. Venezuela, Colombia, Ecuador, Peru, Bolivia, Chile, and Argentina are the seven South American nations that the Andes pass through on their way from north to south.

Only 3rd and 4th rows are correctly matched.

Hence, option b is correct.

11. Consider the following airports:

1. Donyi Polo Airport

2. Kushinagar International Airport

3. Vijayawada International Airport

In the recent past, which of the above have been constructed as Greenfield projects?

(a) 1 and 2 only

(b) 2 and 3 only

(c) 1 and 3 only

(d) 1, 2 and 3

11. Ans: a

Explanation:

A Greenfield project is a type of project that starts from scratch, with no existing infrastructure or resources.

Under the Greenfield Airports policy 2008, the Government of India has accorded ‘In-Principle’ approval for the setting up of 21 Greenfield Airports namely, Mopa in Goa, Navi Mumbai, Shirdi and Sindhudurg in Maharashtra, Kalaburagi, Vijayapura, Hassan and Shivamogga in Karnataka, Dabra (Gwalior) in Madhya Pradesh, Kushinagar and Noida (Jewar) in Uttar Pradesh, Dholera and Hirasar in Gujarat, Karaikal in Puducherry, Dagadarthi, Bhogapuram and Orvakal (Kurnool) in Andhra Pradesh, Durgapur in West Bengal, Pakyong in Sikkim, Kannur in Kerala and Itanagar in Arunachal Pradesh across the country. 

Out of these, 9 Greenfield airports viz. Durgapur, Shirdi, Kannur, Pakyong, Kalaburagi, Orvakal (Kurnool), Sindhudurg, Kushinagar and Donyi Polo, Itanagar have been operationalized.

Donyi Polo Airport is located in Itanagar, Arunachal Pradesh. 

Kushinagar International Airport is located in Uttar Pradesh.

Hence, option a is correct. 

12. With reference to “water vapour”, which of the following statements is/are correct?

1. It is a gas, the amount of which decreases with altitude.

2. Its percentage is maximum at the poles. 

Select the answer using the code given below.

(a) 1 only

(b) 2 only

(c) Both 1 and 2

(d) Neither 1 nor 2

12. Ans: a

Explanation:

Water vapour is a variable gas in the atmosphere, which decreases with altitude. In the warm and wet tropics, it may account for four per cent of the air by volume, while in the dry and cold areas of desert and polar regions, it may be less than one per cent of the air. 

Statement 1 is correct. 

Water vapour also decreases from the equator towards the poles. It also absorbs parts of the insolation from the sun and preserves the earth’s radiated heat. It thus, acts like a blanket allowing the earth neither to become too cold nor too hot. Water vapour also contributes to the stability and instability in the air.

Statement 2 is incorrect.

Hence, option a is correct. 

13. Consider the following description:

1. Annual and daily range of temperatures is low.

2. Precipitation occurs throughout the year.

3. Precipitation varies between 50 cm – 250 cm.

What is this type of climate?

(a) Equatorial climate

(b) China type climate

(c) Humid subtropical climate

(d) Marine West Coast climate

13. Ans: d 

Explanation: 

Marine West Coast Climate: Marine west coast climate is located poleward from the Mediterranean climate on the west coast of the continents. 

The main areas are Northwestern Europe, the west coast of North America, north of California, southern Chile, southeastern Australia and New Zealand. Due to marine influence, the temperature is moderate

and in winter, it is warmer than for its latitude. 

The mean temperature in summer months ranges from 15°-20°C and in winter 4°-10°C. 

The annual and daily ranges of temperature are small.

Precipitation occurs throughout the year. 

Precipitation varies greatly from 50-250 cm.

Hence, option d is correct. 

14. With reference to “Coriolis force”, which of the following statements is/are correct?

1. It increases with an increase in wind velocity.

2. It is maximum at the poles and is absent at the equator.

Select the answer using the code given below:

(a) 1 only

(b) 2 only

(c) Both 1 and 2

(d) Neither 1 nor 2

14. Ans: c

Explanation:

Coriolis Force 

The rotation of the earth about its axis affects the direction of the wind. This force is called the Coriolis force after the French physicist who described it in 1844. It deflects the wind to the right direction in the northern hemisphere and to the left in the southern hemisphere. The deflection is more when the wind velocity is high.

statement 1 is correct.

The Coriolis force is directly proportional to the angle of latitude. It is maximum at the poles and is absent at the equator. 

statement 2 is correct.

Hence, option c is correct. 

15. On June 21 every year, which of the following latitude(s) experience(s) sunlight of more than 12 hours?

1. Equator

2. Tropic of Cancer

3. Tropic of Capricorn

4. Arctic Circle

Select the correct answer using the code given below:

(a) 1 only

(b) 2 only

(c) 3 and 4

(d) 2 and 4

15. Ans: d

Explanation:

UPSC Prelims Answer Key 2024 With Explanation 1

In the summer (June), the daylight increases as we go poleward. At the Arctic Circle, the sun never ‘sets’ at mid-summer (June 21) and there is a complete 24-hour period of continuous daylight. In summer, the region north of the Arctic Circle is popularly referred to as the ‘Land of the Midnight Sun’. At the North Pole, there will be six months of continuous daylight.

After the March equinox the sun appears to move north and is vertically overhead at the Tropic of Cancer on about 21 June. This is known as the June or summer solstice when the northern hemisphere will have its longest day and shortest night.

Hence, option d is correct.

16. One of the following regions has the world’s largest tropical peatland, which holds about three years worth of global carbon emissions from fossil fuels; and the possible destruction of which can exert detrimental effect on the global climate. Which one of the following denotes that region?

(a) Amazon Basin

(b) Congo Basin

(c) Kikori Basin

(d) Rio de la Plata Basin

16. Ans: b

Explanation: 

The Congo Basin is home to the world’s largest tropical peatlands, along with Brazil and Indonesia. The peat swamp forest of the Congo Basin stores around 29 billion tons of carbon – approximately equivalent to three years’ worth of global greenhouse gas emissions – while the Basin as a whole absorbs nearly 1.5 billion tons of carbon dioxide a year. The Basin stretches across six countries- Cameroon, Central African Republic, Democratic Republic of the Congo, Congo, Equatorial Guinea and Gabon. 

Hence, option b is correct. 

17. With reference to perfluoroalkyl and polyfluoroalkyl substances (PFAS) that are used in making many consumer products, consider the following statements:

1. PFAS are found to be widespread in drinking water, food and food packaging materials.

2. PFAS are not easily degraded in the environment.

3. Persistent exposure to PFAS, can lead to bioaccumulation in animal bodies.

Which of the statements given above are correct?

(a) 1 and 2 only

(b) 2 and 3 only·

(c) 1 and 3 only

(d) 1, 2 and 318

17. Ans: d

Explanation:

Perfluoroalkyl substances (PFASs) are substances commonly used in the production of various everyday objects, including among others kitchen dishes, cosmetics, or clothes. They penetrate to the environment and living organisms causing disturbances in the functioning of many internal organs and systems.

Per- and polyfluoroalkyl substances (PFAS) are a complex & ever-expanding group of manufactured chemicals widely used in everyday products. They are called ‘Forever Chemicals’ due to their non-degradability, long-term environmental persistence, and accumulation potential.

Hence, statement 2 is correct.

PFAS are used in many things we use daily and have extensive industrial and consumer applications, including non-stick cookware, food packaging, cleaning solutions, fire-fighting foam, stain/water resistant fabrics, and personal care items. PFAS compounds are considered ‘Contaminants of Emerging Concern’ as exposure to these has been associated with adverse human health issues such as reduced immune response, hormonal imbalances, low birth weight, etc., and even carcinogenic.

The presence of PFAS in drinking water due to the various environmental and health issues described above and in the environment is an urgent global public health issue that needs tremendous attention.

PFAS are used in the aerospace, automotive, construction, and electronics industries. Over time, PFAS may leak into the soil, water, and air.

statement 1 is correct.

People are most likely exposed to these chemicals by consuming PFAS-contaminated water or food, using products made with PFAS, or breathing air containing PFAS. 

Multiple health effects associated with PFAS exposure have been identified and are supported by different scientific studies. Concerns about the public health impact of PFAS have arisen for the following reasons:

  • Widespread occurrence: Studies find PFAS in the blood and urine of people, and scientists want to know if it causes health problems.
  • Numerous exposures: PFAS are used in hundreds of products globally, with many opportunities for human exposure.
  • Growing numbers: PFAS are a group of nearly 15,000 synthetic chemicals, according to a chemicals database (CompTox) maintained by the U.S. Environmental Protection Agency.
  • Persistent: PFAS remain in the environment for an unknown amount of time.
  • Bioaccumulation: People may encounter different PFAS chemicals in various ways. Over time, people may take in more of the chemicals than they excrete, a process that leads to bioaccumulation in bodies.

Statement 3 is correct.

18. Consider the following:

1. Carabid beetles

2. Centipedes

3. Flies                                                                                                                                                                                 

4. Termites

5. Wasps

Parasitoid species are found in how many of the above kinds of organisms?

(a) Only two

(b) Only three

(c) Only four

(d) All five

18. Ans: c

Explanation:

Parasitoid

Most parasitoids — parasitic insects that kill their hosts — live freely and independently as adults; they are lethal and dependent only in their immature stages. Parasitoids can be specialists, targeting either a single host species or several related species, or they can be generalists, attacking many types of hosts. Typically, they attack hosts larger than themselves, eating most or all of their hosts’ bodies before pupating inside or outside them.

UPSC Prelims Answer Key 2024 With Explanation

Centipedes are generalist predators, meaning they eat a wide variety of prey. They eat most anything that is small enough and soft-bodied, which includes insects and other small animals. Because they eat almost any other small organism, including many common pests, centipedes are considered beneficial.

Hence, option c is correct. 

19. Consider the following plants:

1. Groundnut

2. Horse-gram

3. Soybean

How many of the above belong to the pea family?

(a) Only one

(b) Only two

(c) All three

(d) None

19. Ans: c

Explanation:

Fabaceae is pea family of flowering plants (angiosperms), within the order Fabales. 

Some of the most important commercial species include soybeans (Glycine max), garden peas (Pisum sativum), peanuts (Arachis hypogaea), and alfalfa (Medicago sativa). 

Hence, option c is correct. 

20. Consider the following statements:

Statement-I: The Indian Flying Fox is placed under the “vermin” category in the Wild Life (Protection) Act, 1972.

Statement II: The Indian Flying Fox feeds on the blood of other animals.

Which one of the following is correct in respect of the above statements?

(a) Both Statement-I and Statement-II are correct and Statement-II explains Statement-I

(b) Both Statement-I and Statement-II are correct, but Statement-II does not explain Statement-I

(c) Statement-I is correct, but Statement-II is incorrect

(d) Statement-I is incorrect, but Statement-II is correct

20. Ans: c 

Explanation:

Wildlife Protection Act defines vermin – “vermin” means any wild animal specified in Schedule V. 

SCHEDULE V of Wild Life Protection Act 

 1. Common crow 

 2. Fruit bats 

 4. Mice 

 5. Rats 

Hence, statement 1 is correct.

The flying fox, also known as fruit bats or Pteropus gigante is listed as a ‘vermin’ in the Indian Wildlife (Protection) Act 1972, due to its destructive tendencies towards fruit farms.

The Indian flying fox is one of the largest species of bats in the world. Its wing length is 1.2-1.5m. It survives only on fruits.

Hence, statement 2 is incorrect.

21. The organisms “Cicada, Froghopper and Pond skater” are:

(a) Birds

(b) Fish

(c) Insects

(d) Reptiles

21. Ans: c

Explanation: 

Cicada, froghopper and pond skater all belong to the category of insects.

Hence, option c is correct. 

22. Consider the following statements:

Statement-I: Many chewing gums found in the market are considered a source of environmental pollution.

Statement-II: Many chewing gums contain plastic as gum base.

Which one of the following is correct in respect of the above statements?

(a) Both Statement-I and Statement-II are correct and Statement-II explains Statement-I

(b) Both Statement-I and Statement-II are correct, but Statement-II does not explain Statement-I

(c) Statement-I is correct, but Statement-II is incorrect

(d) Statement-I is incorrect, but Statement-II is correct

22. Ans: a

Explanation:

The earliest forms of gum were plant-based resins extracted from trees, most of the sticks we chew today are composed of synthetic petroleum derivatives – plastics. They include; butyl rubber, polythyene and polyvinyl acetate, which are also used in diesel, plastic bags and glue, respectively. These ingredients are often disguised under the name “gum base” on labels, and these plastics provide gum with chewiness.

Chewing gum contributes 100,000 tonnes of plastic pollution annually. An estimated 80-90% of chewing gum is not disposed of properly, making it the one of the most littered items on streets

Due to its plastic contents, chewing gum is non-biodegradable

Both statements 1 & 2 are correct, and statement 2 correctly explains statement 1.

23. Consider the following pairs:

CountryAnimals found in its natural habitat
I.BrazilIndri
2.IndonesiaElk
3.MadagascarBonobo

How many of the pairs given above are correctly matched?

(a) Only one

(b) Only two

(c) All three

(d) None

23. Ans: d

Explanation:

Indris are the largest lemurs in the world. Like all lemurs, indris (Indri indri) are endemic to the island of Madagascar rainforests.

Elk, (Cervus elaphus canadensis), the largest and most advanced subspecies of red deer (Cervus elaphus), found in North America and in high mountains of Central Asia. It is a member of the deer family. 

Wild bonobos can only be found in forests south of the Congo River in the Democratic Republic of Congo (DRC). Bonobos and chimpanzees look very similar and both share 98.7% of their DNA with humans—making the two species our closest living relatives. Bonobos are usually a bit smaller, leaner, and darker than chimpanzees. 

None of the pairs are correctly matched, Hence, option d is correct. 

24. Consider the following statements regarding World Toilet Organization:

1. It is one of the agencies of the United Nations.

2. World Toilet Summit, World Toilet Day and World Toilet College are the initiatives of this organization, to inspire action to tackle the global sanitation crisis.

3. The main focus of its function is to grant funds to the least developed countries and developing countries to achieve the end of open defecation.

Which of the statements given above is/are correct?

(a) 2 only

(b) 3 only

(c) 1 and 2

(d) 2 and 3

24. Ans: a

Explanation:

The World Toilet Organization (WTO) is a global non-profit established in 2001, committed to enhancing sanitation and advocating for safe and hygienic toilets worldwide. 

Through collaborative partnerships with governments, NGOs, and private sector entities, the WTO implements innovative sanitation projects, promotes best practices, and empowers communities to meet their sanitation needs.

Hence, statements 1 and 3 are incorrect.

Among Organisation’s notable achievements are: 

(a) The establishment of UN World Toilet Day, an initiative spearheaded in collaboration with the Singapore government to raise awareness about the global sanitation crisis and the importance of toilets in promoting health, well-being, and human dignity. Recognized by the United Nations General Assembly in 2013, November 19th is now designated as World Toilet Day, aimed at catalyzing action to address the sanitation crisis.

(b) The World Toilet Summit has been the primary toilet-based and sanitation-driven conference in the world, with its most recent summit being held in November 2019, in São Paulo – Brazil. With 18-and-counting annual summits organised in different countries around the world, it encourages delegates to share knowledge, and best practices in the Water, Sanitation and Hygiene (WASH) sector and form potential partnerships across each summit every year. 

(c) World Toilet College was launched in 2016 in Rishikesh in partnership with local partners Global Interfaith Wash Alliance (GIWA) and support of Reckitt Benckiser, one of the largest FMCG companies. 

Hence, statement 2 is correct.

25. Consider the following statements:

1. Lions do not have a particular breeding season.

2. Unlike most other big cats, cheetahs do not roar.

3. Unlike male lions, male leopards do not proclaim their territory by scent marking.

Which of the statements given above are correct?

(a) 1 and 2 only

(b) 2 and 3 only

(c) 1 and 3 only

(d) 1, 2 and 3

25. Ans: b

Explanation:

The mating season for lions is June to October. 

The Gir National Park spread across 1,412 sq km, was recently closed from June 16 to October 15 as part of lions mating season. 

Hence, statement 1 is incorrect.

Unlike other big cats, including lions, tigers, leopards, and jaguars, cheetahs don’t roar. They growl when there is danger, and usually only chirp, purr and meow.

Hence, statement 2 is correct.

Scent marking allows carnivores to communicate with each other by leaving their scent at prominent landmarks such as termite mounds and large trees in savannah ecosystems. These carnivores deposit their scent via scratching, rubbing, urinating, or defecating and often return to the same sites.

Such olfactory cues are used to mark territory, advertise dominance or reproductive status, and alert predators. 

Lions, Leopards and cheetahs will also ‘Sharpen their claws’ on the bark of a tree. Along with keeping their claws in good condition, there are glands in between their digits that leave their scent behind. The same glands deposit scent onto the ground and grass when the animal scratches their feet through the grass. 

Hence, statement 3 is correct.

26. Which one of the following is the correct description of “100 Million Farmers”?

(a) It is a platform for accelerating the transition towards food and water systems that are net-zero (carbon), nature-positive and that aims to increase farmer resilience.

(b) It is an international alliance and a network of individuals and farming organisations interested in supporting and strengthening the development of the organic animal husbandry.

(c) It is a digital platform fully integrated with service providers and built on blockchain that lets buyers, sellers and third parties trade, fertilizers quickly and securely.

(d) It is a platform with the mission of encouraging the farmers to form Farmer Product Organisations or Agribusiness Consortiums, thus facilitating the access to global open markets to sell their products.

26. Ans: a

Explanation: 

100 Million Farmers is a platform supporting private and public leaders to position food and farmers as central pillars of the global climate and nature agenda and accelerate collective action to scale climate- and nature-friendly agricultural practices.

100 Million Farmers has bold objectives for accelerating the transition towards food and water systems that are net-zero, nature-positive, and that increase farmer resilience. All three objectives are critical to achieving a successful transition. 

Hence, option a is correct. `

27. Consider the following:

1. Battery storage

2. Biomass generators

3. Fuel cells

4. Rooftop solar photovoltaic units

How many of the above are considered “Distributed Energy Resources”?

(a) Only one

(b) Only two

(c) Only three

(d) All four

27. Ans: d

Explanation: 

Distributed energy resources are small, modular, energy generation and storage technologies that provide electric capacity or energy where you need it. 

DER systems can be made up of one or more primary technologies—such as internal combustion engines, combustion turbines, fuel cells, photovoltaics, wind turbines, and batteries.

DER technologies include wind turbines, photovoltaics (PV), fuel cells, microturbines, reciprocating engines, combustion turbines, cogeneration, and energy storage systems.

Examples of distributed energy resources that can be installed include:

  • rooftop solar photovoltaic units
  • wind generating units
  • battery storage
  • batteries in electric vehicles used to export power back to the grid
  • combined heat and power units, or tri-generation units that also utilise waste heat to provide cooling
  • biomass generators, which are fuelled with waste gas or industrial and agricultural by-products.
  • open and closed cycle gas turbines
  • reciprocating engines (diesel, oil)
  • hydro and mini-hydro schemes
  • fuel cells.

Hence, option d is correct. `

28. Which one of the following shows a unique relationship with an insect that has coevolved with it and that is the only insect that can pollinate this tree?

(a) Fig

(b) Mahua

(c) Sandalwood

(d) Silk cotton

28. Ans: a

Explanation:

Figs have an obligate mutualism with tiny fig-pollinating wasps (family Agaonidae). Female wasps enter receptive syconia, where they pollinate female flowers. They also lay eggs in some flowers, where their larvae induce galls. Some weeks later, the wasp offspring emerge from their galls into the syconium, just as the male flowers have matured their pollen sacs. The new generation of female wasps leave the syconium through holes made by the males and carry pollen to receptive syconia elsewhere. This symbiosis is exploited by many species of fig-parasitic wasps. Most of them do not enter the syconium or contribute to pollination but use long ovipositors to inject eggs through the fig wall so that their larvae can feed on either fig flowers or other wasps.

Their interaction dominates the lives of both and, as each relies on the other for reproduction, 

Figs are selected to raise some seeds (female function) and some wasps (male function as pollen vectors); however, wasps are selected to maximise their own offspring production, with no regard for seed production. This leads to the coevolution of various fig and wasp traits.

Hence, option a is correct.

29. Consider the following:

1. Butterflies

2. Fish

3. Frogs

How many of the above have poisonous species among them?

(a) Only one

(b) Only two

(c) All three

(d) None

29. Ans: c

Explanation: 

Poisonous Butterflies – Pipevine Swallow Tail, The Postman. 

Poisonous Fish – Puffer fish, spotted trunkfish. 

Poisonous Frogs – Golden Poison Dart Frog, Blue Poison Dart Frog, Black-Legged Poison Dart Frog.

Hence, option c is correct. 

30. Consider the following:

1. Cashew

2. Papaya

3. Red sanders

How many of the above trees are actually native to India?

(a) Only one

(b) Only two

(c) All three

(d) None

30. Ans: a

Explanation: 

The Papaya originated in Mesoamerica, likely in southern Mexico 

The cashew is native to northeastern Brazil. Portuguese missionaries took it to East Africa and India during the late 16th century. 

The Red Sanders (Pterocarpus santalinus) is an endemic tree species to the forests of Seshachalam, Veligonda, Lankamala and Palakona Hills range of AndhraPradesh. 

Hence, option a is correct. `

31. With reference to radioisotope thermoelectric generators (RTGs), consider the following statements:

1. RTGs are miniature fission reactors.

2. RTGs are used for powering the onboard systems of spacecraft.

3. RTGs can use Plutonium-238, which is a by-product of weapons development.

Which of the statements given above are correct?

(a) 1 and 2 only

(b) 2 and 3 only

(c) 1 and 3 only

(d) 1, 2 and 3

31. Ans: b

Explanation:

Radioisotope Thermoelectric Generators (RTGs) are lightweight, compact spacecraft power systems. RTGs provide electrical power using heat from the natural radioactive decay of plutonium-238, in the form of plutonium oxide. 

Sometimes referred to as “nuclear batteries,” RTGs are not fission reactors, nor is plutonium the type that is used for nuclear weapons. 

Hence, statement 1 is incorrect & statement 2 is correct.

The fuel in an RPS — short for radioisotope power systems — is plutonium oxide, a radioactive material that produces alpha particles. Alpha particles are a particular type of ionizing radiation that can be shielded by material as thin as a piece of paper. Plutonium-238 is not the type of plutonium used for nuclear weapons and would not work well as fuel in a nuclear reactor.

In 1959, a prototype RTG generated a sustained 2.5 watts of power over 90 days rapid advances in nuclear chemistry — in the U.S., an outgrowth of the World War II-era Manhattan Project that produced the first nuclear weapons — yielded an array of new radioactive materials. One of these, Pu-238, is nearly ideal for fueling RTGs. 

Hence, statement 3 is correct.

32. Consider the following statements:

Statement I: Giant stars live much longer than dwarf stars.

Statement II: Compared to dwarf stars, giant stars have a greater rate of nuclear reactions.

Which one of the following is correct in respect of the above statements?

(a) Both Statement-I and Statement-II are correct and Statement-II explains Statement-I

(b) Both Statement-I and Statement-II are correct, but Statement-II does not explain Statement-I

(c) Statement I is correct, but Statement II is incorrect

(d) Statement I is incorrect, but Statement II is correct

32. Ans: d

Explanation:

There are two kinds of very large stars. Giant stars have masses from eight times to as much as 100 times that of the mass of our sun. These massive stars have hotter and denser cores than dwarf stars (stars that are smaller than five times the mass of our sun). Therefore, giant stars have a greater rate of the nuclear reactions that light up stars. Massive stars also use up the hydrogen fuel in their core faster, despite starting out with much more of it, meaning they live much shorter lives than dwarf stars.

Hence, statement 1 is incorrect & statement 2 is correct.

33. Which one of the following is synthesised in the human body that dilates blood vessels and increases blood flow?

(a) Nitric oxide

(b) Nitrous oxide

(c) Nitrogen dioxide

(d) Nitrogen pentoxide

33. Ans:  a

Explanation:

Nitric oxide is a molecule that’s produced naturally by your body, and it’s important for many aspects of your health.

Its most important function is vasodilation, meaning it relaxes the inner muscles of the blood vessels, causing them to widen and increase circulation.

Nitric oxide production is essential for overall health because it allows blood, nutrients, and oxygen to travel to every part of your body effectively and efficiently.

Hence, option a is correct. 

34. Consider the following activities:

1. Identification of narcotics on passengers at airports or in aircraft

2. Monitoring of precipitation

3. Tracking the migration of animals

In how many of the above activities can the radars be used?

(a) Only one

(b) Only two

(c) All three

(d) None

34. Ans: b

Explanation:

RADAR is an electromagnetic system for the detection and location of target objects such as aircraft, ships, spacecraft, vehicles, people, and the natural environment which can reflect a signal back. It uses electromagnetic radio waves to determine the angle, range, or velocity of objects.

A weather radar can determine the precipitation type (rain, snow, hail, etc.) and spot its location. With the help of a weather radar map, it is also possible to predict where the rain will be moving next and how intense it will be. 

Radar has several benefits over other techniques to study migration such as visual observations, trapping, and banding, as it works well at altitude and over large distances, is unaffected by the absence of light, and is relatively independent of weather conditions.

Trace narcotic detectors offer a vital tool for identifying microscopic chemical signatures from drugs and residues. Tiny samples are analyzed to reveal narcotic evidence even when no drugs are visible.

Trace detectors collect minute samples via methods like air puffs or swabs wiped over surfaces and materials. Advanced instrumentation analyzes each sample for the presence of chemical markers associated with narcotics. Trace residues of drugs, manufacturing chemicals, or cutting agents can be identified even at levels invisible to the naked eye.

Techniques like ion mobility spectrometry (IMS) and mass spectrometry provide rapid yet highly precise identification of narcotic signatures.  IMS analyzes the mobility rate of ionized molecules in an electric field, while mass spec separates compounds by their mass-to-charge ratio.

Both methods can detect drug traces down to pictograms or nanograms. This enables the discovery of clandestine threats that evade standard security screening.

Full-body X-ray scanners:-  X-ray scanners can be an effective tool for detecting narcotics and other hazardous items. Leveraging state-of-the-art neural network algorithms and knowledge of human body physiology, the software visualizes suspicious areas with an indication of the probability of drug and narcotic presence. In a matter of seconds, it analyzes images and outlines potential areas where narcotics and drugs may be concealed.

Hence, option b is correct.

35. Consider the following aircraft:

1. Rafael

2. MiG-29

3. Tejas MK-1

How many of the above are considered fifth-generation fighter aircraft?

(a) Only one

(b) Only two

(c) All three

(d) None

35. Ans: d

Explanation:

Fifth-generation fighter aircraft are the most technologically advanced jets ever built. They were designed to operate with the help of digital programs and fly by wire to counter the threats they faced on the battlefield. Sophisticated avionics, stealth technology, and super maneuverability are just a few advantages over older jets that give them domination in the sky.

few countries have decided to pursue the development of fifth-generation fighter jets:

  • Lockheed F-22 Raptor and F-35 Lighting II – USA
  • Sukhoi T-50 PAK-FA – Russia
  • Chengdu J-20 and Shenyang J-31 Gyrfalcon – China 

Hence, option d is correct.

36. In which of the following are hydrogels used?           

1. Controlled drug delivery in patients

2. Mobile air-conditioning systems

3. Preparation of industrial lubricants 

Select the correct answer using the code given below:

(a) 1 only

(b) 1 and 2 only

(c) 2 and 3 only

(d) 1, 2 and 3

36. Ans: a

Explanation:  

Hydrogel products constitute three-dimensional networks that contain a group of polymeric materials, the hydrophilic structure of which renders them capable of holding large amounts of water. These biomaterials can integrate large quantums of biological fluids and swell. When swelled, they are soft & rubbery and resemble the living tissue, exhibiting excellent biocompatibility. 

Hydrogels are three-dimensional cross-linked polymer networks that are smart enough to respond to fluctuations in environmental stimuli (ionic strength, pH, temp, presence of enzyme, electric field, etc.) and swell or shrink accordingly. In the swollen state, they are soft and rubbery, resembling living tissue and exhibiting excellent biocompatibility. 

Hydrogels are capable of delivering genetically engineered pharmaceuticals, viz. protein and peptides and improve the therapeutic efficacy of drugs.

Application of hydrogels

  • Lab-grown tissues
  • Oxygen diffusers
  • Drug delivery systems 
  • Impact protectors (cushioning)

Hence, statement 1 is correct.

Air conditioners in passenger cars, vans, buses and freight trucks – collectively known as mobile air conditioning (MAC) – consume almost 2 million barrels of oil equivalent per day (Mboe/d).

Mobile air conditioning is part of the vehicle control system, which provides cooling, demisting, and humidity control, as well as heating, defrosting, and air filtering.

HFO-1234yf was selected by almost all vehicle manufacturers as the refrigerant. 

37. Which one of the following is the exhaust pipe emission from Fuel Cell Electric Vehicles, powered by hydrogen?

(a) Hydrogen peroxide

(b) Hydronium

(c) Oxygen

(d) Water vapour

37. Ans: d 

Explanation:

FUEL CELL ELECTRIC VEHICLE 

A fuel cell uses the chemical energy of hydrogen or other fuels to cleanly and efficiently produce electricity. If hydrogen is the fuel, the only products are electricity, water, and heat.

Fuel cell electric vehicles (FCEVs) powered by hydrogen are more efficient than conventional internal combustion engine vehicles. 

Hence, option d is correct.

38. Recently, the term “pumped-storage hydropower” is actually and appropriately discussed in the context of which one of the following?                                                       .

(a) Irrigation of terraced crop fields

(b) Lift irrigation of cereal crops

(c) Long-duration energy storage

(d) Rainwater harvesting system

38. Ans: c

Explanation:

PUMP STORAGE HYDROPOWER 

Pumped storage hydropower (PSH) is a type of hydroelectric energy storage. It is a configuration of two water reservoirs at different elevations that can generate power as water moves down from one to the other (discharge), passing through a turbine. 

Pumped storage hydropower plants store electricity by pumping water up from a lower reservoir to an upper reservoir and then releasing it through turbines when power is needed. They represent 30% of net hydropower additions through 2030 according to the IEA forecast. 

The increasing need in many markets for system flexibility and storage to facilitate the integration of larger shares of variable renewables drives record growth of pumped storage projects between 2021 and 2030. 

Hence, option c is correct.

39. “Membrane Bioreactors” are often discussed in the context of:

(a) Assisted reproductive technologies

(b) Drug delivery nanotechnologies

(c) Vaccine production technologies

(d) Wastewater treatment technologies

39. Ans: d

Explanation:

Membrane Bioreactors (MBRs), have the advantage of combining a suspended growth biological reactor with solids removal via filtration. The membranes can be designed for and operated in small spaces and with high removal efficiency of contaminants such as nitrogen, phosphorus, bacteria, biochemical oxygen demand, and total suspended solids. 

The membrane filtration system in effect can replace the secondary clarifier and sand filters in a typical activated sludge treatment system. 

Membrane filtration allows a higher biomass concentration to be maintained, thereby allowing smaller bioreactors to be used. Membrane filtration involves the flow of water-containing pollutants across a membrane. Water permeates through the membrane into a separate channel for recovery. Because of the cross-flow movement of water and waste constituents, materials left behind do not accumulate at the membrane surface but are carried out of the system for later recovery or disposal. The water passing through the membrane is called the permeate, while the water with the more concentrated materials is called the concentrate or retentate.

A MEMBRANE BIOREACTOR (MBR) process combines a microfiltration or ultrafiltration membrane unit with a suspended growth bioreactor and is now widely used in both municipal and industrial Wastewater Treatment Plants.

Hence, option d is correct.

40. With reference to the Indian economy, “Collateral Borrowing and Lending Obligations” are the instruments of:

(a) Bond market

(b) Forex market

(c) Money market

(d) Stock market

40. Ans: c

Explanation:

Collateralized Borrowing and Lending Obligations (CBLOs)

The Clearing Corporation of India Ltd. (CCIL) has developed and introduced with effect from January 20, 2003 a money market instrument called Collateralised Borrowing and Lending Obligation (CBLO)

Hence, option c is correct. 

41. The total fertility rate in an economy is defined as:

(a) the number of children born per 1000 people in the population in a year.

(b) the number of children born to a couple in their lifetime in a given population.

(c) the birth rate minus the death rate.

(d) the average number of live births a woman would have by the end of her child-bearing age.

41. Ans: d

Explanation:

Total Fertility Rate is defined as the average number of children that would be born to a woman if she experiences the current fertility pattern throughout her reproductive span (15-49 years).

Hence, option d is correct.

42. Consider the following statements:

1. In India, Non-Banking Financial Companies can access the Liquidity Adjustment Facility window of the Reserve Bank of India.

2. In India, Foreign Institutional Investors can hold Government Securities (G-Secs).

3. In India, Stock Exchanges can offer separate trading platforms for debts.

Which of the statements given above is/are correct?

(a) 1 and 2 only

(b) 3 only

(c) 1, 2 and 3

(d) 2 and 3 only

42. Ans: d

Explanation:

SCHEME OF LIQUIDITY ADJUSTMENT FACILITY: 

Under the scheme, (i) Repo auctions (for absorption of liquidity) and (ii) Reverse Repo auctions (for injection of liquidity) will be conducted on a daily basis 

Eligibility

All Scheduled Commercial Banks (excluding Regional Rural Banks) and Primary Dealers (PDs) having Current Account and Subsidiary General Ledger account (SGL) with RBI, Mumbai will be eligible to participate in the Repo and Reverse Repo auctions. 

Hence, statement 1 is incorrect.

G-Secs market – Major players in the G-Secs market include commercial banks and PDs besides institutional investors like insurance companies. PDs play an important role as market makers in the G-Secs market. Other participants include co-operative banks, regional rural banks, mutual funds, provident and pension funds. 

Foreign Portfolio Investors (FPIs) are allowed to participate in the G-Secs market within the quantitative limits prescribed from time to time.

Hence, statement 2 is correct.

Corporates also buy/ sell the G-Secs to manage their overall portfolio. 

As advised by SEBI, the stock exchanges (like NSE, BSE, and MCX) have been asked to create dedicated debt segments in their trading platforms. In compliance with this, stock exchanges have launched a debt trading (G-Secs as also corporate bonds) segment which generally caters to the needs of retail investors. 

Hence, statement 3 is correct.

Hence, option d is correct.

43. In India, which of the following can trade in Corporate Bonds and Government Securities?

1. Insurance Companies

2. Pension Funds

3. Retail Investors

Select the correct answer using the code given below:

(a) 1 and 2 only

(b) 2 and 3 only

(c) 1 and 3 only

(d) 1, 2 and 3

43. Ans: d

Explanation:

Corporate bonds are debt securities issued by private and public corporations. Companies issue corporate bonds to raise money for a variety of purposes, such as building a new plant, purchasing equipment, or growing the business. When one buys a corporate bond, one lends money to the “issuer,” the company that issued the bond. In exchange, the company promises to return the money, also known as “principal,” on a specified maturity date. Until that date, the company usually pays you a stated rate of interest, generally semiannually. While a corporate bond gives an IOU from the company, it does not have an ownership interest in the issuing company, unlike when one purchases the company’s equity stock. 

A Government Security (G-Sec) is a tradeable instrument issued by the Central Government or the State Governments. It acknowledges the Government’s debt obligation. Such securities are short-term (usually called treasury bills, with original maturities of less than one year) or long-term (usually called Government bonds or dated securities with an original maturity of one year or more). In India, the Central Government issues both, treasury bills and bonds or dated securities while the State Governments issue only bonds or dated securities, which are called the State Development Loans (SDLs). G-Secs carry practically no risk of default and, hence, are called risk-free gilt-edged instruments. 

G-Secs market – Major players in the G-Secs market include commercial banks and PDs besides institutional investors like insurance companies. PDs play an important role as market makers in the G-Secs market. Other participants include co-operative banks, regional rural banks, mutual funds, provident and pension funds. 

Foreign Portfolio Investors (FPIs) are allowed to participate in the G-Secs market within the quantitative limits prescribed from time to time.

Corporates also buy/ sell the G-Secs to manage their overall portfolio. 

Under the retail direct scheme, individuals can invest in the G-Secs market.

Hence, option d is correct. 

44. Consider the following:

1. Exchange-Traded Funds (ETF)

2. Motor vehicles

3. Currency swap

Which of the above is/are considered financial instruments?

(a) 1 only

(b) 2 and 3 only

(c) 1, 2 and 3

(d) 1 and 3 only

44. Ans: d

Explanation:

A financial instrument is any contract that gives rise to a financial asset of one entity and a financial liability or equity instrument of another entity.  

A contract or agreement representing a tradable asset, liability, or equity instrument in financial markets is a financial instrument.

Classification of Financial Instruments: –

Financial instruments can be divided into several types, each with a specific function.  

Cash Instruments

These assets are easily convertible into cash and have a fixed value. Currency notes, bank deposits, and money market securities like Treasury bills are a few examples.

Debt Instruments

The loan an investor makes to an issuer in exchange for recurring interest payments; debt instruments represent the principal repayment. Examples include loans, mortgages, corporate debentures, and bonds.

Equity Instruments

Equity instruments reflect ownership in a business or enterprise and grant the holder voting rights and a share of the profits. Examples are equity mutual funds, Exchange-traded Funds, common stocks, and preferred stocks.

Derivative Instruments

An underlying asset, such as commodities, bonds, stocks, or currencies, gives derivatives their value. Examples include future contracts, swaps, options, and forwards.

Money Market Instruments

These financial instruments can borrow and lend money in the money market. They have a short-term maturity. Some examples are treasury bills and repurchase agreements.

Hence, option d is correct.

45. With reference to the sectors of the Indian economy, consider the following pairs:

Economic activitySector
1.Storage of agricultural produceSecondary
2.Dairy farmPrimary
3.Mineral explorationTertiary
4.Weaving clothSecondary

How many of the pairs given above are correctly matched?

(a) Only one

(b) Only two

(c) Only three

(d) All four

45. Ans: b

Explanation:

Economic ActivitySector
1.Storage of agricultural produceTertiary
2.Dairy farmPrimary
3.Mineral explorationPrimary
4.Weaving clothSecondary

Primary Sector – Activities that are undertaken by directly using natural resources. Take, for example, the cultivation of cotton. It takes place within a crop season. For the growth of the cotton plant, we depend mainly, but not entirely, on natural factors like rainfall, sunshine and climate. The product of this activity, cotton, is a natural product. Similarly, in the case of an activity like dairy, we are dependent on the biological process of the animals and the availability of fodder etc. The product here, milk, also is a natural product. Similarly, minerals and ores are also natural products. When we produce a good by exploiting natural resources, it is an activity of the primary sector. 

Why primary? 

This is because it forms the base for all other products that we subsequently make. Since most of the natural products we get are from agriculture, dairy, fishing, and forestry, this sector is also called agriculture and related sectors. 

Secondary Sector – Activities in which natural products are changed into other forms through ways of manufacturing that we associate with industrial activity. It is the next step after the primary. The product is not produced by nature but has to be made and therefore some process of manufacturing is essential. This could be in a factory, a workshop or at home. For example, using cotton fibre from the plant, we spin yarn and weave cloth. Using sugarcane as a raw material, we make sugar or gur. We convert earth into bricks and use bricks to make houses and buildings. Since this sector gradually became associated with the different kinds of industries that came up, it is also called as industrial sector. 

Tertiary Sector – These are activities that help in the development of the primary and secondary sectors. These activities, by themselves, do not produce a good but they are an aid or a support for the production process. For example, goods that are produced in the primary or secondary sector would need to be transported by trucks or trains and then sold in wholesale and retail shops. At times, it may be necessary to store these in godowns. We also may need to talk to others over the telephone or send letters (communication) or borrow money from banks (banking) to help production and trade. Transport, storage, communication, banking, and trade are some examples of tertiary activities. Since these activities generate services rather than goods, the tertiary sector is also called the service sector.

Only 2nd and 4th pairs are correctly matched.

Hence, option b is correct.

46. Consider the following materials:

1. Agricultural residues

2. Corn grain

3. Wastewater treatment sludge

4. Wood mill waste

Which of the above can be used as feedstock for producing Sustainable Aviation Fuel?

(a) 1 and 2 only

(b) 3 and 4 only

(c) 1, 2, 3 and 4

(d) 1, 3 and 4 only

46. Ans: c

Explanation:

Menu of Sustainable feedstock for producing Sustainable Aviation Fuel are:

  1. Corn grain
  2. Oil seeds
  3. Algae
  4. Other fats, oils, and greases
  5. Agricultural residues
  6. Forestry residues
  7. Wood mill waste
  8. Municipal solid waste streams
  9. Wet wastes (manures, wastewater treatment sludge)
  10. Dedicated energy crops.

Hence, option c is correct.

47. With reference to physical capital in the Indian economy, consider the following pairs:

ItemsCategory
     1.Farmer’s ploughWorking capital
     2.ComputerFixed capital
     3.Yarn used by the weaverFixed capital
     4.PetrolWorking capital

How many of the above pairs are correctly matched

(a) Only one

(b) Only two

(c) Only three

(d) All four

47. Ans: b

Explanation:

ItemsCategory
1.Farmer’s ploughFixed capital
2.ComputerFixed capital
3.Yarn used by the weaverWorking capital
4.PetrolWorking capital

The financial needs of a business can be categorised as follows: 

(a) Fixed capital requirements: In order to start a business, funds are required to purchase fixed assets like land and buildings, plant and machinery, and furniture and fixtures 

This is known as the fixed capital requirements of the enterprise.

farmer’s plough, computer would be part of fixed capital

(b) Working capital requirements: An enterprise’s financial requirements do not end with the procurement of fixed assets. No matter how small or large a business is, it needs funds for its day-to-day operations. This is known as working capital, which is used for holding current assets such as stock of material and bills receivables and for meeting current expenses like salaries, wages, taxes, and rent. 

yarn used by weavers, petrol would be part of the working capital

Only 2nd and 4th pairs are correctly matched.

Hence, option b is correct.

48. Which one of the following words/phrases is, most appropriately used to denote “an interoperable network of 3D virtual worlds that can be accessed simultaneously by millions of users, who can exert property rights over virtual items”?

(a) Big data analytics

(b) Cryptography

(c) Metaverse

(d) Virtual matrix

48. Ans: c

Explanation:

The metaverse implies a shared environment that spans a multitude of 3D virtual worlds. Participants in the metaverse will be able to move freely through these virtual worlds, taking their identities, entitlements and goods with them. At least that’s one vision of how a universe of 3D virtual worlds will work.

Hence, option c is correct.

49. With reference to the rule/rules imposed by the Reserve Bank of India while treating foreign banks, consider the following statements:

1. There is no minimum capital requirement for wholly-owned banking subsidiaries in India.

2. For wholly owned banking subsidiaries in India, at least 50% of the board members should be Indian nationals.

Which of the statements given above is/are correct?

(a) 1 only

(b) 2 only

(c) Both 1 and 2

(d) Neither 1 nor 2

49. Ans: b

Explanation:

Eligibility for setting up a wholly-owned subsidiary (WOS)

a) Setting up of WOS by a foreign bank in India should have the approval of the home country regulator/supervisor.

b) A foreign bank applying for setting up a WOS in India must satisfy RBI that it is subject to adequate prudential supervision as per internationally accepted standards, which includes consolidated supervision in its home country.

Minimum capital requirement

a) The initial minimum paid-up voting equity capital for a wholly-owned subsidiary (WOS) shall be Rs. 5 billion.

b) The newly set up wholly-owned subsidiary (WOS) of the foreign bank would be required to bring in the entire amount of initial capital upfront, which should be funded by free foreign exchange remittance from its parent.

c) The WOS shall meet the Basel III requirements on a continuous basis from the time. 

Hence, statement 1 is incorrect.

Corporate governance of wholly-owned subsidiary (WOS)

The composition of the board of directors of WOS should meet the following requirements:

a) Not less than 51 per cent of the total number of members of the board of directors shall consist of persons as defined under Section 10A of the Banking Regulation Act, 1949;

b) Not less than two-third of the directors should be non-executive directors;

c) Not less than one-third of the directors should be independent of the management of the subsidiary in India, its parent and any subsidiary or other associate of the foreign bank parent;

d) Not less than 50 per cent of directors should be Indian nationals/NRIs/PIOs subject to the condition that one-third of the directors are Indian nationals resident in India;

e) WOS’s of foreign banks will have a Part-time Chairman and full-time Chief Executive Officer (CEO)

Hence, statement 2 is correct.

50. With reference to Corporate Social Responsibility (CSR) rules in India, consider the following statements:

1. CSR rules specify that expenditures that benefit the company directly or its employees will not be considered as CSR activities.

2. CSR rules do not specify minimum spending on CSR activities.

Which of the statements given above is/are correct?

(a) 1 only

(b) 2 only

(c) Both 1 and 2

(d) Neither 1 nor 2

50. Ans:  a

Explanation:

Corporate Social Responsibility 

Rule 2(d) of the Companies (CSR) Rules, 2014 defines the term CSR as follows – “Corporate Social Responsibility (CSR)” means the activities undertaken by a Company in pursuance of its statutory obligation laid down in section 135 of the Act in accordance with the provisions contained in these rules, but shall not include the following, namely:-

(i) Activities undertaken in pursuance of the normal course of business of the company:

ii) Any activity undertaken by the company outside India except for training of Indian sports personnel representing any State or Union territory at national level or India at international level; 

(iii) Contribution of any amount directly or indirectly to any political party under section 182 of the Act;

(iv) Activities benefitting employees of the company as defined in clause (k) of section 2 of the Code on Wages, 2019 (29 of 2019);

(v) Activities supported by the companies on a sponsorship basis for deriving marketing benefits for its products or services;

(vi) Activities carried out for the fulfilment of any other statutory obligations under any law in force in India;

Hence, statement 1 is correct.

As per the Companies Act, 2013, a Company covered under Section 135(1) of the Companies Act, 2013 is required to spend 2% of the Average net profits of the company made during the three immediately preceding financial years towards Corporate Social Responsibility (CSR)

Hence, statement 2 is incorrect.

51. Consider the following statements:

Statement-I: If the United States of America (USA) were to default in its debt, holders of US Treasury Bonds will not be able to exercise their claims to receive payment.

Statement-II: The USA Government debt is not backed by any hard assets, but only by the faith of the Government.

Which one of the following is correct in respect of the above statements?

(a) Both Statement-I and Statement-II are correct and Statement-II explains Statement-I

(b) Both Statement-I and Statement-II are correct, but Statement-II does not explain Statement-I

(c) Statement I is correct, but Statement II is incorrect

(d) Statement-I is incorrect, but Statement-II is correct

51. Ans: d

Explanation:

What is an Unsecured bond?

Unsecure bonds do not have specific collateral. These bonds are based solely on the creditworthiness of the issuer.

Investors in unsecured bonds depend on the financial strength and performance of the issuer to meet payment obligations.

Unsecured bonds are highly risky because there is no specific asset to be seized upon default. Investors in unsecured bonds rely solely on the repayment capacity of the issuer. 

To compensate for the increased risk, issuers of unsecured bonds typically offer higher interest rates

Unsecured bonds rely more on credit ratings because they lack collateral.

In the event of default, holders of unsecured bonds tend to have weaker positions.

Examples of unsecured bonds include corporate bonds, where investors rely on the financial strength and creditworthiness of the company, and government bonds, which depend on the creditworthiness of the issuing government.  

In the case of American Treasury Bonds – The Treasury Bond holders both domestic and foreign can 

sue the American government.

American debt holders can sue in the United States District Court and United States Court of Federal Claims.

Foreign debt holders can sue in their own Countrys court system, but not in United States court to redress the United States government’s failure to make good on debt payments.

However, recovery of payment may not be possible in either case because there is no specific asset to be seized upon default, i.e. holders of unsecured bonds tend to have weaker positions.

Hence, option d is correct.

52. Consider the following statements:

Statement-I: Syndicated lending spreads the risk of borrower default across multiple lenders.

Statement II: The syndicated loan can be a fixed amount/lump sum of funds, but cannot be a credit line.

Which one of the following is correct in respect of the above statements?

(a) Both Statement-I and Statement-II are correct and Statement-II explains Statement-I

(b) Both Statement-I and Statement-II are correct, but Statement-II does not explain Statement-I

(c) Statement I is correct, but Statement II is incorrect

(d) Statement I is incorrect, but Statement II is correct

52. Ans:  c

Explanation:

Syndicated loans started as a way of allowing lenders to lend large sums of money to a single borrower, where the sums involved went far beyond the credit appetite of a single lender.

An investment-grade syndicated loan is usually a multi-lender transaction, where lenders (usually banks) contract with a borrower to provide a loan on common terms and conditions governed by a common document (or sets of documents).

A syndicated loan is offered by a group of lenders who work together to provide credit to a large borrower. The borrower can be a corporation, an individual project, or a government.

Each lender in the syndicate contributes part of the loan amount, and they all share in the lending risk.

Hence, statement 1 is correct.

What is a Credit Line?

Credit Line is an amount of money a person or company is allowed to borrow during a particular period of time from one or more financial organizations. 

Syndicated loans can be a fixed amount or a credit line for a particular period. 

Hence, statement 2 is incorrect.

53. Consider the following statements in respect of the digital rupee:

1. It is a sovereign currency issued by the Reserve Bank of India (RBI) in alignment with its monetary policy.

2. It appears as a liability on the RBI’s balance sheet.

3. It is insured against inflation by its very design.

4. It is freely convertible against commercial bank money and cash.

Which of the statements given above are correct?

(a) 1 and 2 only

(b) 1 and 3 only

(c) 2 and 4 only

(d) 1, 2 and 4

53. Ans: d

Explanation: 

A Digital Rupee or Central Bank Digital Currency (CBDC) is a sovereign currency issued by the central bank, Reserve Bank of India with respect to its monetary policy.

Hence, statement 1 is correct.

CBDC will be a sovereign currency available in digital form. Like normal currency issued by the central bank, CBDC will also appear as a liability on the central bank’s balance sheet. 

Hence, statement 2 is correct. Features of Digital Rupee

  • The Digital Rupee is issued by the Reserve Bank of India and is legally recognized as a secure form of payment accepted by individuals, businesses, and governmental bodies.
  • The issuance follows the central bank’s financial policies 
  • Holders have the freedom to convert the digital Rupee into physical cash through commercial banks.
  • Legal Tender: CBDCs are considered legal tender, usable for all types of transactions.
  • Central Bank Control: CBDCs are controlled and regulated by the central bank, ensuring stability and trustworthiness.
  • Programmable Money: CBDCs can have programmable features, such as smart contracts, enabling automated, self-executing financial agreements.
  • It is a fungible legal tender for which individuals will not need a bank account to store

Hence, statement 4 is correct.

Reserve Bank of India – Liabilities and Assets

LIABILITIES  ASSETS
1. Notes Issued

Notes in Circulation
Notes held in Banking Department
1. Foreign currency Assets
2. Gold Coin and Bullion
3. Rupee Securities (including Treasury Bills)
2. Deposits

Central Government
Market Stabilization Scheme
State Governments
Scheduled Commercial Banks
Scheduled State Co-operative Banks
Other Banks
4. Loans and Advances

Central Government
State Governments
NABARD
Scheduled Commercial Banks
Scheduled State Co-op. Banks
Industrial Development Bank of India
Export-Import Bank of India
5. Bills Purchased and Discounted
Commercial
Treasury
6. Investments

54. With reference to ancient India, Gautama Buddha was generally known by which of the following epithets?

1. Nayaputta

2. Shakyamuni

3. Tathagata

Select the correct answer using the code given below:

(a) 1 only

(b) 2 and 3 only

(c) 1, 2 and 3

(d) None of the above are epithets of Gautama Buddha

54. Ans: b

Explanation: 

Epithets of Lord Buddha

Buddha- the Awakened One, the Enlightened One, Sakyamuni (the Sakya sage), Sakyasimha (the Sakya Lion), Sugata (the Happy One), Satthar, Sattha/Shastar (the Teacher), Jina (the Conqueror), Bhagavat (the Blessed One), Lokanatha (the Lord of the World), Sarvajna (the Omniscient One), Dharma- raja (the King of Truth), Tathagata (one who has thus gone)

Epithets of Lord Mahavira

Muni, Mahana, Nayaputta, Vira, Mahavira and Bhagavan were the main epithets for Lord Mahavira. 

Hence, option b is correct. 

55. Consider the following information:

Archaeological SiteStateDescription
1.ChandraketugarhOdishaTrading Port town
2.InamgaonMaharashtraChalcolithic site
3.MangaduKeralaMegalithic site
4.SalihundamAndhra PradeshRock-cut cave shrines

In which of the above rows is the given information correctly matched?

(a) 1 and 2

(b) 2 and 3

(c) 3 and 4

(d) 1 and 4

55. Ans: b

Explanation:

Archaeological SiteStateDescription
1.ChandraketugarhWest Bengal Important centre of trade
2.InamgaonMaharashtraChalcolithic site
3.MangaduKerala Megalithic site
4.SalihundamAndhra PradeshBuddhist Stupa

Chandraketugarh:- It is located in the Ganga delta, in the 24 Parganas district of West Bengal, about 25 miles northeast of Kolkata.

In early historic times, Chandraketugarh was connected to the Ganga by the Vidyadhari river and must have been an important centre of trade, and possibly also a political centre.  Chandraketugarh was a major centre of   terracotta craft. 

Inamgaon:-  (in Pune district) is located on a terrace of the God, a tributary of the Bhima. It is one of the largest, most intensively and extensively excavated Chalcolithic sites in Maharashtra.

The excavations, undertaken by a team from Deccan College, Pune, under the direction of M.K. Dhavalikar, H. D. Sankalia, and Z.D. Ansari, lasted for 12 seasons between 1968 and 1983 and provided a lot of information about the lives of the farmers who lived in this place hundreds of years ago. Period I (c. 1600–1400 BCE) belonged to the Malwa culture, Period II (c. 1400–1000 BCE) to the early Jorwe culture,  and Period III (c. 1000–700 BCE) to the late Jorwe culture.

Mangadu: Megaliths are widely distributed in South India. In Tamil Nadu, the sites include Adichanallur, Amritamangalam, Kunnattur, Sanur, Vasudevanallur, Tenkasi, Korkai, Kayal, Kalugumalai, Perumalmalai, Pudukkotai, Tirukkampuliyar, and Odugat-tur. 

Important sites in Kerala include Pulimattu, Tengakkal, Cenkotta, Muthukar, Peria Kanal, Machad, Pazhayannur, and Mangadu. 

The megaliths at Mangadu in the Kollam district of Kerala date from c. 1000–100 BCE. 

Among the important megalithic sites in Karnataka are Brahmagiri, Maski, Hanamsagar, Terda l-Halingali,  T. Narsipur, and Hallur, Kumarnahalli.

Sites in Andhra include Kadambapur, Nagarjunakonda, Yelleswaram, Gallapalli, Tadapatri, Mirapuram, and Amaravati. 

Salihundam, and Ramathirtham in Andhra are reputed for Buddhist Stupas.

Only 2nd   and 3rd rows are correctly matched 

Hence, option b is correct. 

56. Who of the following rulers of medieval India gave permission to the Portuguese to build a fort at Bhatkal?

(a) Krishnadevaraya

(b) Narasimha Saluva

(c) Muhammad Shah III

(d) Yusuf Adil Shah

56. Ans: a 

Explanation: 

In 1509, Alfonso de Albuquerque became Governor of Portuguese in India.

Krishnadevaraya became king in 1509 and was virtually the master of the Entire South India. There were several quasi-independent vassals under Krishnadevaraya – The Chiefs of Srirangapatnam, Barkapur, Garsopa, Calicut, Bhatkal and Barkur. 

In 1510 Albuquerque captured Goa under Adil Shah. On the success of Albuquerque at Goa, Krishnadevaraya granted permission for the Portuguese to build a Fort at Bhatkal for the protection of Portuguese Trade. 

Hence, option a is correct. 

57. With reference to revenue collection by Cornwallis, consider the following statements:

1. Under the Ryotwari Settlement of revenue collection, the peasants were exempted from revenue payment in case of bad harvests or natural calamities.

2. Under the Permanent Settlement in Bengal, if the Zamindar failed to pay his revenues to the state, on or before the fixed date, he would be removed from his Zamindari.

Which of the statements given above is/are correct?

(a) 1 only

(b) 2 only

(c) Both 1 and 2

(d) Neither 1 nor 2

57. Ans: b

Explanation:

Munro introduced the Ryotwari system

Under this system, the cultivator was recognized as the owner of the land and was responsible for the collection of land revenue. This system was introduced in the early 19th century in the Madras and Bombay presidencies. The system was revised periodically usually after 20 to 30 years when the revenue demand was usually increased.

Peasants were tenants in the eyes of the government for which they paid rent and not tax. The land revenue was fixed at exorbitant rates and thus peasants were left with barely enough for their sustenance. The ryots had to pay in times even when their produce was destroyed by floods or when drought prevailed. The ultimate aim behind the ryotwari system was to consolidate the Company’s government in the south by expanding its revenue base. It has been argued that a significant social upheaval was brought about by the Ryotwari Settlement which greatly harmed the status held by village headmen and resulted in the outbreak of the Deccan riots.

Hence, statement 1 is incorrect. 

Cornwallis began the Permanent Settlement in Bengal and Bihar (1793). John Shore, the Record Keeper opined that Zamindars were the owners of the land with regard to revenue settlement and Cornwallis complied with this view. The Decennial settlement was declared permanent in the year 1793 with approval from the Court of Directors and Zamindars were given due recognition as landowners. 

Zamindars were given orders to pay 89 per cent of annual revenue and the rest was the zamindar’s share.

The Zamindars had to pay a fixed amount of revenue by the due date and before sunset (known as the sunset law). This provision of the settlement frightened the zamindars, and turned them into paupers because if they failed in being punctual, their zamindari were sold off.

Hence, statement 2 is correct.

58. Consider the following statements:

1. There are no parables in the Upanishads.

2. Upanishads were composed earlier than the Puranas.

Which of the statements given above is/are correct?

(a) 1 only

(b) 2 only

(c) Both 1 and 2

(d) Neither 1 nor 2

58. Ans: b

Explanation:

The Vedas are generally considered to have two portions- the Karma-Kanda (portion dealing with action or rituals) and Jnana-Kanda (portion dealing with knowledge). 

The Samhita and the Brahmanas represent mainly the Karma-Kanda or the ritual portion, while the Upanishads chiefly represent the Jnana-Kanda or the knowledge portion. 

The Upanishads are often called ‘Vedanta’. Literally, Vedanta means the end of Veda, Vedasaya antah, the conclusion (Anta) as well as the goal (Anta) of the Vedas.

The word ‘Upanishad’ has been derived from the root Sad (to sit), to which are added two prefixes: Upa and Ni. The prefix Upa denotes nearness and Ni totality. Thus, this word means ‘sitting nearby devotedly.’

The Upanishads contain parables. Some of the important parables are Yama—Nachiketa, the story of Jabala-Satyakama, Satyakama and Upakoshala, the story of Rishi Yajnavalkya, Narada—-Sanatkumara, Indra-Virochana and Prajapati, Ushasti Chakrayana, etc. 

Hence, statement 1 is incorrect.

Period of Composition of some important Ancient Indian Text.

  • Vedic Text – 2000 – 500 BCE 
  • Vedanga Text – 600 – 200 BCE 
  • Mahabharata – 400 BCE – 400 CE
  • Ramayana – 500 BCE – 300 CE 
  • Puranas – 100 BCE – 600 CE
  • Dharmasutras – 600 BCE – 300 BCE 
  • Smritis – 200 BCE – 900 CE
  • Tripitaka – 600 BCE – 300 BCE 
  • Jaina Canon – 500 BCE – 500 CE 
  • Sangam Texts – 300 BCE – 300 CE
  • Trikkural – 500 CE – 600 CE 
  • Tamil epics (Silppadikaram and Manimekalai) – 500 CE – 600 CE 

Hence, statement 2 is correct.

59. Consider the following statements:

1. India is a member of the International Grains Council.

2. A country needs to be a member of the International Grains Council for exporting or importing rice and wheat.

Which of the statements given above is/are correct?

(a) 1 only

(b) 2 only

(c) Both 1 and 2

(d) Neither 1 nor 2

59. Ans: a

Explanation:

INTERNATIONAL GRAINS COUNCIL 

The International Grains Council (IGC) is an intergovernmental organization that seeks to further international cooperation in grains trade, promote expansion, openness and fairness in the grains sector; contribute to grain market stability and to enhance world food security. Grains, rice and oilseeds market conditions are monitored on a daily basis and made available through daily reports and web-based information services. Data monitored includes market information (price index) as well as export prices of grains (wheat, maize, barley, soybeans, rice) and oilseeds, global supply and demand, freight rates, and five-year global projections. 

Member countries of the International Grain Council

Argentina, Australia, Canada, European Union, India, Kazakhstan, Russian Federation, Serbia, Turkey, Ukraine, United States, Algeria, Côte d’Ivoire, Cuba, Egypt, Iran, Iraq, Japan, Kenya, South Korea, Morocco, Norway, Oman, Pakistan, Saudi Arabia, South Africa, Switzerland, Tunisia, Vatican City

Hence, statement 1 is correct.

A country need not be a member of the International Grains Council for exporting or importing rice and wheat. 

For example – The major destinations of India’s non-basmati white rice exports include Thailand, Italy, Spain, Sri Lanka and the USA. The importing countries from India like Sri Lanka, Thailand, Spain, and Italy are not members of the International Grain Council.

Hence, statement 2 is incorrect. 

60. Which one· of the following was the latest inclusion in the Intangible Cultural Heritage List of UNESCO?

(a) Chhau dance

(b) Durga puja

(c) Garba dance

(d) Kumbhmela

60. Ans: c

Explanation:

UNESCO has inscribed the popular Gujarati folk dance Garba on its Representative List of the Intangible Cultural Heritage of Humanity in 2023. 

A ritualistic and devotional dance, Garba is particularly celebrated for nine days during the festival of Navaratri. The festival is dedicated to the worship of the feminine energy, Shakti. Garba takes place within homes and temple courtyards, public spaces in villages, urban squares, streets, and large open grounds. Thus, it becomes an all-encompassing participatory community event.

Hence, option c is correct. 

In 2021 Durga Puja in Kolkata became Representative List of the Intangible Cultural Heritage of Humanity. 

In 2017, Kumbh Mela became a Representative List of the Intangible Cultural Heritage of Humanity. 

In 2010, the Chhau dance became a Representative List of the Intangible Cultural Heritage of Humanity.

Source: https://ich.unesco.org/en/state/india-IN?info=elements-on-the-lists

61. Who was the Provisional President of the Constituent Assembly before Dr Rajendra Prasad took over?

(a) C. Rajagopalachari

(b) Dr. B.R. Ambedkar

(c) T.T. Krishnamachari

(d) Dr. Sachchidananda Sinha

61. Ans: d

Explanation: 

The Constitution was framed by the Constituent Assembly of India, established by the members of the provincial assemblies elected by the people of India. Dr Sachidanand Sinha was the first president of the Constituent Assembly. Later, Dr Rajendra Prasad was elected its president.

Hence, option d is correct.

62. With reference to the Government of India Act, 1935, consider the following statements:

1. It provided for the establishment of an All-India Federation based on the union of the British Indian Provinces and Princely States.

2. Defence and Foreign Affairs were kept under the control of the federal legislature.

Which of the statements given above is/are correct?

(a) 1 only

(b) 2 only

(c) Both 1 and 2

(d) Neither 1 nor 2

62. Ans: d

Explanation:

Government of India Act 1935 

PART II. THE FEDERATION OF INDIA

CHAPTER I. ESTABLISHMENT OF FEDERATION AND ACCESSION OF INDIAN STATES. 

5.-(1) It shall be lawful for His Majesty if an address on that behalf has been presented to him by each House of Parliament and if the condition hereinafter mentioned is satisfied, to declare by Proclamation that as from the day therein appointed there – shall be united in a Federation under the Crown, by the name of the Federation of India –

(a) the Provinces hereinafter called Governors’ Provinces; and

(b) the Indian States which have acceded or may thereafter accede to the Federation 

Source: https://www.legislation.gov.uk/ukpga/1935/2/pdfs/ukpga_19350002_en.pdf

The Act envisaged an all-India Federation which was to consist of 11 Governor’s Provinces, 6 Chief Commissioner’s Provinces and such Indian States as would agree to join the federation.

Source: https://interstatecouncil.gov.in/wp-content/uploads/2015/06/CHAPTERI.pdf

The Government of India Act, of 1935 which, among others, envisaged a ‘federation of all-India’, consisting of the British provinces and the Indian states willing to join it.

Source: https://www.presidentofindia.gov.in/pranab_mukherjee/speeches/speech-president-india-shri-pranab-mukherjee-delivers-first-bhairon-singh-shekhawat

By the Government of India Act, 1935 which was enacted by the British Parliament on 2 August 1935, India was established as a federation comprising of the Governors’ Provinces, Chief Commissioners’ Provinces and the Indian States which had or would accede to the Federation of India

Source: https://main.sci.gov.in/pdf/LU/article_370.pdf

The Government of India act 1935 provided for an All India federation According to the act of 1935 all the Provinces were to join the Indian federation automatically. Entry into the federation was to be purely voluntary action on the part of each state, however small and insignificant the state may be. At the time of joining the federation, the ruler of the state was to execute an Instrument of accession in favour of the crown.

Hence, statement 1 is incorrect. 

The Act of 1935 also provided for dyarchy at the centre. That dyarchy which was abolished in the provinces, was introduced at the centre, and certain federal subjects were reserved in the hands of the Governor-General to be administered by him with the assistance of not more than 3 councillors to be appointed by him. Those subjects were Defence, External Affairs, Ecclesiastical Affairs and the Administration of the Tribal Areas. In the Administration of other federal subjects, the Governor General was to be aided and advised by a council of Ministers whose number was not to exceed 10. The federal Ministry was to be to administer all the federal departments except the above-mentioned reserved Departments. 

Hence, statement 2 is incorrect.

63. Which one of the following is a work attributed to playwright Bhasa?

(a) Kavyalankara

(b) Natyashastra

(c) Madhyama-vyayoga

(d) Mahabhashya

63. Ans: c

Explanation:

Bhasa wrote several dramas including the Pancharatra, Dutavakya, Balacharita, and Svapna-Vasavadatta, Pratima-natak. 

Bhasa was a playwright who wrote works such as the Madhyama-vyayoga, Duta-Ghatotkacha, Dutavakya, Balacharita, and Charudatta.

Hence, option c is correct.

64. Sanghabhuti, an Indian Buddhist monk, who travelled to China at the end of the fourth century AD, was the author of a commentary on:

(a) Prajnaparamita Sutra

(b) Visuddhimagga

(c) Sarvastivada Vinaya

(d) Lalitavistara

64. Ans: c

Explanation:

Many Indian monks travelled to China. They included Sanghabhuti, author of a commentary on the Sarvastivada Vinaya, who was in China in 381–84 CE. 

Other Indian monks or monks of Indian origin who travelled to China are Kumarajiva (5th century), Paramartha (6th century), and Bodhidharma (6th century). 

Monk Punyatrata travelled to central Asia along with his student Dharmayashas and translated several Sarvastivadin texts between 397 and 401 CE. 

Hence, option c is correct.

65. Consider the following properties included in the World Heritage List released by UNESCO:

1. Shantiniketan

2. Rani-ki-Vav

3. Sacred Ensembles of the Hoysalas

4. Mahabodhi Temple Complex at Bodhgaya

How many of the above properties were included in 2023?

(a) Only one

(b) Only two

(c) Only three

(d) All four

65. Ans: b

Explanation:

The Committee of UNESCO World Heritage Site added 42 new sites (33 cultural, 9 natural) to the UNESCO World Heritage List, bringing the total to 1199 (993 cultural, 227 natural, 39 mixed).

Indian Properties included in the World Heritage List released by UNESCO for 2023 

  1. Shantiniketan
  2. Sacred Ensembles of the Hoysalas

Hence, option b is correct.

66. As per Article 368 of the Constitution of India, the Parliament may amend any provision of the Constitution by way of:

1. Addition

2. Variation

3. Repeal

Select the correct answer using the code given below:

(a) 1 and 2 only

(b) 2 and 3 only

(c) 1 and 3 only

(d) 1, 2 and 3

66. Ans: d

Explanation:

Article 368 in the Constitution of India 

368: Power of Parliament to amend the Constitution and procedure, therefore

(1) Notwithstanding anything in this Constitution, Parliament may in the exercise of its constituent power amend by way of addition, variation or repeal any provision of this Constitution in accordance with the procedure laid down in this article.

However, the Parliament cannot amend those provisions which form the ‘basic structure’ of the Constitution.

Hence, option d is correct.

67. Consider the following countries:

1. Italy

2. Japan

3. Nigeria

4. South Korea

5. South Africa

Which of the above countries are frequently mentioned in the media for their low birth rates, or ageing population or declining population?

(a) 1, 2 and 4

(b) 1, 3 and 5

(c) 2 and 4 only

(d) 3 and 5 only

67. Ans: a

Explanation:

South Korea -The country’s fertility rate is the lowest in the world, with the UN warning in 2021 that South Korea was on track to halve its population by 2100.

Japan: Low birth rates, long life

With the world’s second-largest ageing population, Japan is also contending with a low birth rate, leading the country’s Prime Minister Fumio Kishida to announce in January 2023 that “Japan is standing on the verge of whether we can continue to function as a society”.

Italy: A pact for senior citizens

Close to a quarter of Italy’s population is aged over 65, More recently, Prime Minister Giorgia Meloni signed a “Pact for senior citizens”, a draft law “preventing elderly people from being ‘parked’ in healthcare facilities”

Hence, option a is correct.

68. Which of the following statements are correct in respect of a Money Bill in the Parliament?

1. Article 109 mentions special procedure in respect of Money Bills.

2. A Money Bill shall not be introduced in the Council of States.

3. The Rajya Sabha can either approve the Bill or suggest changes but cannot reject it.

4. Amendments to a Money Bill suggested by the Rajya Sabha have to be accepted by the Lok Sabha.

Select the answer using the code given below:

(a) 1 and 2 only

(b) 2 and 3 only

(c) 1, 2 and 3

(d) 1, 3 and 4

68. Ans: c

Explanation:

MONEY BILL IN THE PARLIAMENT

Article 109 in the Constitution of India

109. Special procedure in respect of Money Bills.—       

(1) A Money Bill shall not be introduced in the Council of States.  

Hence, statements 1 and 2 are correct. 

A Money Bill can be introduced only in the Lok Sabha. After it has been passed by Lok Sabha, it is transmitted to Rajya Sabha for its recommendations and that House is, within a period of fourteen days from the date of the receipt of the Bill, required to return the Bill to Lok Sabha with its recommendations, if any. 

The Lok Sabha may either accept or reject all or any of the recommendations made by Rajya Sabha. If Lok Sabha accepts any of the recommendations made by Rajya Sabha, the Bill is deemed to have been passed by both the Houses with the amendments recommended by Rajya Sabha and accepted by Lok Sabha. 

If, however, Lok Sabha does not accept any of the recommendations of Rajya Sabha, the Money Bill is deemed to have been passed by both the Houses of Parliament in the form in which it was passed by Lok Sabha without any of the amendments recommended by Rajya Sabha.

 If the Rajya Sabha does not return the Bill within the prescribed period of fourteen days, the Bill is deemed to have been passed by both the Houses of Parliament at the expiry of the period in the form in which it was passed by the Lok Sabha. 

Hence, statement 3 is correct and 4 is incorrect.

69. Which of the following is/are correctly matched in terms of equivalent rank in the three services of Indian Defence forces?

ArmyAirforceNavy
  1.BrigadierAir CommodoreCommander
  2.Major GeneralAir Vice MarshalVice Admiral
  3.MajorSquadron LeaderLieutenant Commander
  4.Lieutenant ColonelGroup CaptainCaptain

Select the correct answer using the code given below:

(a) 1 and 4

(b) 1 and 3

(c) 2, 3 and 4

(d) 3 only

69. Ans: d

Explanation:

Number of years of serviceIndian ArmyIndian Air ForceIndian Navy
Field MarshalMarshal of AirForceAdmiral of Fleet
By SelectionGeneralAir Chief MarshalAdmiral
28 years by selectionLt. GeneralAir MarshalVice Admiral
25 years by selectionMajor GeneralAir Vice MarshalRear Admiral
23 years by selectionBrigadierAir CommodoreCommodore
After 26 years (15 years by Selection)ColonelGroup CaptainCaptain
After 13 yearsLt. ColonelWing CommanderCommander
After 6 yearsMajorSquadron LeaderLt. Commander
After 2 yearsCaptainFlight LieutenantLieutenant
After 18 months of trainingLieutenantFlying OfficerSub Lieutenant

Hence, option d is correct. 

70. The North Eastern Council (NEC) was established by the North Eastern Council Act, 1971. Subsequent to the amendment of the NEC Act in 2002, the Council comprises which of the following members?

1. Governor of the Constituent State

2. Chief Minister of the Constituent State

3. Three Members to be nominated by the President of India

4. The Home Minister of India

Select the correct answer using the code given below:

(a) 1, 2 and 3 only

(b) 1, 3 and 4 only

(c) 2 and 4 only

(d) 1, 2, 3 and 4

70. Ans: a

Explanation:

North Eastern Council (NEC) was constituted as a statutory advisory body under the NEC Act 1971

The NEC Act was amended by the Parliament in 2002 (Act No. 68 of 2002). 

The ‘North Eastern Areas’ now means the area comprising the 8 States of Assam, Manipur, Meghalaya, Nagaland, Tripura, Arunachal Pradesh, Sikkim and Mizoram. 

The Council now has as its members 

  • the Governors of the 8 States mentioned above, 
  • the Chief Ministers of the said States and 
  • three Members nominated by the President. 

The President will nominate the Chairman of the Council and he need not be nominated from amongst the other Members. 

Hence, option a is correct.

71. How many Delimitation Commissions have been constituted by the Government of India till December 2023?

(a) One

(b) Two

(c) Three

(d) Four

71. Ans: d

Explanation:

Delimitation literally means the act or process of fixing limits or boundaries of territorial constituencies in a country or a province having a legislative body. The job of delimitation is assigned to a high-power body. Such a body is known as the Delimitation Commission or a Boundary Commission. In India, such

Delimitation Commissions have been constituted 4 times – in 1952 under the Delimitation Commission Act, 1952, in 1963 under the Delimitation Commission Act, 1962, in 1973 under the Delimitation Act, 1972 and in 2002 under the Delimitation Act, 2002. 

The Delimitation Commission in India is a high-power body whose orders have the force of law and cannot be challenged in any court. These orders come into force on a date to be specified by the President of India on this behalf. Copies of its orders are laid before the House of the People and the State Legislative Assembly concerned, but no modifications are permissible therein by them.

Source: https://www.eci.gov.in/delimitation

Hence, option d is correct.

72. The Constitution (71st Amendment) Act, 1992 amends the Eighth Schedule to the Constitution to include which of the following languages?

1. Konkani

2. Manipuri

3. Nepali

4. Maithili

Select the correct answer using the code given below:

(a) 1, 2 and 3

(b) 1, 2 and 4

(c) 1, 3 and 4

(d) 2, 3 and 4

72. Ans: a

Explanation:

The Eighth Schedule to the Constitution consists of 22 languages: – 

Of these 14 were initially included in the Constitution

These included Assamese, Bengali, Gujarati, Hindi, Kannada, Kashmiri, Malayalam, Marathi, Oriya, Punjabi, Sanskrit, Tamil, Telugu and Urdu.

The Constitution (Twenty-first Amendment) Act, 1967, amended the Eighth Schedule to the Constitution so as to include Sindhi as one of the languages, thereby raising the total number of languages listed in the schedule to fifteen. 

The 71st Amendment, enacted in 1992, included three more languages Konkani, Manipuri and Nepali to the list. 

In 2003, the 92nd Amendment was enacted, which added Bodo, Dogri, Santhali and Maithili, raising the total number of languages to 22.   

Demands of languages for inclusion in the Eighth Schedule

At present, there are demands for inclusion of 38 more languages in the Eighth Schedule to the Constitution. These are:- (1) Angika, (2) Banjara, (3) Bazika, (4) Bhojpuri, (5) Bhoti, (6) Bhotia, (7) Bundelkhandi (8) Chhattisgarhi, (9) Dhatki, (10) English, (11) Garhwali (Pahari), (12) Gondi, (13) Gujjar/Gujjari (14) Ho, (15) Kachachhi, (16) Kamtapuri, (17) Karbi, (18) Khasi, (19) Kodava (Coorg), (20) Kok Barak, (21) Kumaoni (Pahari), (22) Kurak, (23) Kurmali, (24) Lepcha, (25) Limbu, (26) Mizo (Lushai), (27) Magahi, (28) Mundari, (29) Nagpuri, (30) Nicobarese, (31) Pahari (Himachali), (32) Pali, (33) Rajasthani, (34) Sambalpuri/Kosali, (35) Shaurseni (Prakrit), (36) Siraiki, (37) Tenyidi and (38) Tulu.   

Hence, option a is correct.

73. Consider the following pairs:

PartyIts Leader
1.Bharatiya Jana SanghDr.Shyama Prasad Mukherjee
2.Socialist PartyC. Rajagopalachari
3.Congress for DemocracyJagjivan Ram
4.Swatantra PartyAcharya Narendra Dev

How many of the above are correctly matched?

(a) Only one

(b) Only two

(c) Only three

(d) All four

73. Ans: b

Explanation:

PARTY    LEADER
Bharatiya Jana SanghDr. Shyama Prasad Mukherjee
Swatantra Party  C. Rajagopalachari
Congress for Democracy Jagjivan Ram
Congress Socialist Party          Acharya Narendra Dev

The Socialist Party is an independent incarnation of the Congress Socialist Party (CSP) formed in 1934 within the Congress. The CSP was founded by Jaiprakash Narayan, Acharya Narendra Dev, Dr Ram Manohar Lohia. It merged itself into the Janata Party in 1977 immediately after the independence.

Only 1st and 3rd pairs are correctly matched.

Hence, option b is correct.

74. Which of the following statements are correct about the Constitution of India?

1. Powers of the Municipalities are given, in Part IX A of the Constitution.

2. Emergency provisions are given in Part XVIII of the Constitution.

3. Provisions related to the amendment of the Constitution are given in Part XX of the Constitution.

Select the answer using the code given below:

(a) 1 and 2 only

(b) 2 and 3 only

(c) 1 and 3 only

(d) 1, 2 and 3

74. Ans: d

Explanation:

Parts   Subject Matter          Articles Covered
  1.IX A    The Municipalities   243-P to 243-ZG
   2.XVIII           Emergency Provisions  352 to 360
3.XX  Amendment of the Constitution   368

Hence, option d is correct.      

75. Which one of the following statements is correct as per the Constitution of India?

(a) Inter-State trade and commerce is a State subject under the State List.

(b) Inter-State migration is a State subject under the State List.

(c) Inter-State quarantine is a Union subject under the Union List.

(d) Corporation tax is a State subject under, the State List.

75. Ans: c  

Explanation:

Union List 

Entry 42. Inter-State trade and commerce   

Entry 81. Inter-state migration; inter-state quarantine.

Entry 85. Corporation tax          

Hence, option c is correct.

76. Under which of the following Articles of the Constitution of India, has the Supreme Court of India placed the Right to Privacy?

(a) Article 15

(b) Article 16

(c) Article 19

(d) Article 21

76. Ans: d

Explanation: 

Article 21 in the Constitution of India

Article 21- Protection of life and personal liberty.—No person shall be deprived of his life or personal liberty except according to procedure established by law.  

The Constitution does not grant in specific and express terms any right to privacy as such. Right to Privacy is not enumerated as a Fundamental Right in the Constitution. However, such a right has been culled by the Supreme Court from Article 21 and several other provisions of the Constitution read with the Directive Principles of State Policy.

In the Kharak Singh vs. State of Uttar Pradesh,1963, the minority opinion by Justice Subba Rao was in favour of inferring the Right to Privacy from the expression ‘personal liberty’ in Article 21.

In Govind vs. State of Madhya Pradesh, the Supreme Court accepted a limited Fundamental Right to Privacy “as an emanation” from Articles 19(a), (d) and 21. The Right to Privacy is not, however, absolute; reasonable restrictions can be placed thereon in public interest under Article 19(5).

Again, in R Rajagopal vs. State of Tamil Nadu, the Supreme Court has asserted that Right to Privacy is implicit in the Right to Life and Liberty guaranteed to the citizens by Article 21. It is a “right to be let alone”.

In People’s Union for Civil Liberties vs. Union of India Supreme Court observed that: “We have, therefore, no hesitation in holding that right to privacy is a part of the right to life and personal liberty enshrined under Article 21 of the Constitution. The said right cannot be curtailed ‘except according to procedure established by law.

Hence, option d is correct.

77. What are the duties of the Chief of Defence Staff (CDS) as Head of the Department of Military Affairs?

1. Permanent Chairman of Chiefs of Staff Committee

2. Exercise military command over the three Service Chiefs

3. Principal Military Advisor to Defence Minister on all tri-service matters

Select the correct answer using the code given below:

(a) 1, 2 and 3

(b) 1 and 2 only

(c) 2 and 3 only

(d) 1 and 3 only

77. Ans: d

Explanation:

The Cabinet Committee on Security on 24th December 2019 took the historic decision to create the post of the Chief of Defence Staff (CDS) to enhance the quality of Military Advice to Political Leadership through integration of Service inputs. 

The duties and functions of the Chief of Defence Staff (CDS) include the following:

  1. To head the Department of Military Affairs in Ministry of Defence and function as its Secretary.
  2. To act as the Principal Military Advisor to Hon’ble Raksha Mantri on all Tri-Service matters.
  3. To function as the Permanent Chairman of the Chiefs of Staff Committee
  4. To administer the Tri-Service organizations/agencies/commands.
  5. To be a member of Defence Acquisition Council chaired by Hon’ble Raksha Mantri.
  6. To function as the Military Advisor to the Nuclear Command Authority.
  7. To bring about jointness in operation, logistics, transport, training, support services, communications, repairs and maintenance, etc of the three Services.
  8. To ensure optimal utilisation of infrastructure and rationalise it through jointness among the Services.
  9. To implement Five-Year Defence Capital Acquisition Plan and Two-Year roll-on Annual Acquisition Plans, as a follow up of Integrated Capability Development Plan.
  10. To assign inter-services prioritisation to capital acquisition proposals based on the anticipated budget.
  11. To bring about reforms in the functioning of three Services with the aim to augment combat capabilities of the Armed Forces by reducing wasteful expenditure.

Hence, statements 1 &3 are correct.

CDS will not exercise any military command, including over the three Service Chiefs, to provide impartial advice to the political leadership.

Hence, statement 2 is incorrect.

Former Chief of the Army Staff, General Bipin Rawat was appointed as the country’s first Chief of Defence Staff on 31st December 2019. 

78. Operation undertaken by the Army towards the upliftment of the local population in remote areas to include addressing of their basic needs is called: 

(a) Operation Sankalp

(b) Operation Maitri

(c) Operation Sadbhavana

(d) Operation Madad

78. Ans: c

Explanation:

Indian Army launched Operation Sadbhavana in 1998 to extend a helping hand in rebuilding the socio-economic life of people. 

Operation Sadbhavana is aimed at helping the people to help themselves. Under it the Army acts as a facilitator and catalyst for development projects that are identified jointly with the state administration and the people. Efforts are concentrated on projects at the village level. 

The Army actively assists in planning, provides technical assistance, makes available specialized equipment and conducts supervision.  Maximum use is made of local labour, craftsmen and material so that the money spent on the projects is pumped into the local economy. Completion of projects in a time bound frame is the priority. 

The projects of the Operation Sadbhavana have been connected with providing quality education, women empowerment, better healthcare and community development. 

Hence, option c is correct.

79. The longest border between any two countries in the world is between:

(a) Canada and the United States of America

(b) Chile and Argentina

(c) China and India

(d) Kazakhstan and Russian Federation

79. Ans: a

Explanation:

The international land border between the United States and Canada is the longest in the world at almost 8,900 kilometers. It includes the border between Canada and the continental U.S. as well as the border between Alaska and northern Canada

Length of longest international land borders worldwide:

CountriesLength (in kilometers)
1.Canada-United States8893
2.Kazakhstan-Russia7644
3.Argentina-Chile6691
4.China-Mongolia4630
5.Bangladesh-India4142
6.China-Russia4133
7.Mongolia-Russia3452
8.Bolivia-Brazil3403
9.India-Pakistan3190
10.Mexico-United States3155

Hence, option a is correct.

80. Which of the following statements about the Ethics Committee in the Lok Sabha are correct?

1. Initially it was an ad-hoc Committee.

2. Only a Member of the Lok Sabha can make a complaint relating to unethical conduct of a member of the Lok Sabha.

3. This Committee cannot take up any matter which is sub-judice.

Select the answer using the code given below:

(a) 1 and 2 only

(b) 2 and 3 only

(c) 1 and 3 only

(d) 1, 2 and 3

80. Ans: c

Explanation:

The Speaker 13th Lok Sabha (Late Shri G.M.C. Balayogi) constituted the first Ethics Committee in Lok Sabha on 16 May, 2000 with the following terms of reference namely:- 

(a) to oversee the moral and ethical conduct of the Members; and 

(b) to examine the cases referred to it with reference to ethical and other misconduct of the Members.  

Ethics Committee even today continues to be an ad-hoc Committee of the House.

Hence, statement 1 is correct.

Procedure for ethics complaints:

Any person or member may make a complaint relating to unethical conduct of a member of Lok Sabha. Provided that if a complaint is made by any person, it shall be forwarded by a member. 

Hence, statement 2 is incorrect.

The Committee on Ethics shall not take up any matter which is sub-judice and the decision of the Committee as to whether such matter is or is not sub-judice shall for the purposes of these rules be treated as final. 

Hence, statement 3 is correct.

81. Consider the following statements regarding ‘Nari Shakti Vandan Adhiniyam’:

1. Provisions will come into effect from the 18th Lok Sabha.

2. This will be in force for 15 years after becoming an Act.

3. There are provisions for the reservation of seats for Scheduled Castes Women within the quota reserved for the Scheduled Castes.

Which of the statements given above are correct?

(a) 1, 2 and 3

(b) 1 and 2 only

(c) 2 and 3 only

(d) 1 and 3 only

81. Ans: c

Explanation:

Nari Shakti Vandan Adhiniyam or the Women’s Reservation Bill (now known as One-hundred and Sixth Amendment Act) was passed in the Parliament. The bill seeks to provide 33% reservation to women in Lok Sabha and State Legislative Assemblies.

Reservation for women will not be implemented in 2024 Lok Sabha poll; it can come into effect only after a delimitation based on a census conducted after the Bill has been passed; next delimitation in 2026. Effectively, this means that the earliest implementation of the women’s quota in the Lok Sabha can be in the 2029 general election, rather than 18th Lok Sabha general election.

Hence, statement 1 is incorrect.

The Bill reserves, as nearly as may be, one-third of all seats for women in Lok Sabha, state legislative assemblies, and the Legislative Assembly of the National Capital Territory of Delhi.  This will also apply to the seats reserved for SCs and STs in Lok Sabha and states legislatures.  

The reservation will be provided for a period of 15 years. However, it shall continue till such date as determined by a law made by Parliament. 

Hence, statements 2 & 3 are correct.

82. Which of the following statements about ‘Exercise Mitra Shakti-2023’ are correct?

1. This was a joint military exercise between India and Bangladesh.

2. It commenced in Aundh (Pune).

3. Joint response during counter-terrorism operations was a goal of this operation.

4. Indian Air Force was a part of this exercise.

Select the answer using the code given below:

(a) 1, 2 and 3

(b) 1, 2 and 4

(c) 1, 3 and 4

(d) 2, 3 and 4

82. Ans: d

Explanation:

The ninth edition of INDIA – SRI LANKA Joint Military exercise “Exercise MITRA SHAKTI-2023” was held in Aundh (Pune).  The exercise is being conducted from 16th to 29th November 2023.

Hence, statement 1 is incorrect and statement 2 is correct.

The Indian contingent, of 120 personnel, is being represented mainly by troops from the MARATHA LIGHT INFANTRY Regiment.

The Sri Lankan side is being represented by personnel from 53 Infantry Division. 

15 personnel from Indian Air Force and five personnel from Sri Lankan Air Force are also participating in the exercise.

Hence, statement 4 is correct.

The aim of the exercise is to jointly rehearse conduct of Sub Conventional operations under Chapter VII of United Nations Charter. The Scope of the exercise includes synergising joint responses during counter-terrorist operations.

Hence, statement 3 is correct.

83. A Writ of Prohibition is an order issued by the Supreme Court or High Courts to:

(a) a government officer prohibiting him from taking a particular action.

(b) the Parliament/Legislative Assembly to pass a law on Prohibition.

(c) the lower court prohibiting continuation of proceedings in a case.

(d) the Government prohibiting it from following an unconstitutional policy.

83. Ans: c

Explanation:

The jurisdiction for the grant of a writ of prohibition is primarily supervisory, and the object of the writ is to restrain courts or inferior tribunals from exercising jurisdiction which they do not possess at all or to prevent them from exceeding the limits of their jurisdiction. The object is to confine courts or tribunals of inferior or limited jurisdiction within their bounds. Therefore, the writ of prohibition is issued to prohibit the tribunal from making an ultra vires order or decision. This writ is available during the pendency of the proceedings. It is not available against the public officer who is not vested with the judicial functions. Where excess of jurisdiction is apparent on the face of proceedings, a writ of prohibition is not a matter of discretion but may be had as a right. A writ of prohibition may be issued not only in case of absence or excess of jurisdiction but also in cases where the court or tribunal assumes jurisdiction under a law, which itself contravenes some of the fundamental rights guaranteed by the Constitution.

Hence, option c is correct.

84. Consider the following statements:

1. It is the Governor of the State who recognizes and declares any community of that State as a Scheduled Tribe.

2. A community declared as a Scheduled Tribe in a State need not be so in another State.

Which of the statements given above is/are correct?

(a) 1 only

(b) 2 only

(c) Both 1 and 2

(d) Neither 1 nor 2

84. Ans: b

Article 342 in the Constitution of India 

(1) The President may, with respect to any State or Union territory, and where it is a State after consultation with the Governor thereof, by public notification specify the tribes or tribal communities or parts of or groups within tribes or tribal communities which shall for the purposes of this Constitution be deemed to be Scheduled Tribes in relation to that State or Union territory, as the case may be.

(2) Parliament may by law include in or exclude from the list of Scheduled Tribes specified in a notification issued under clause (1) any tribe or tribal community or part of or group within any tribe or tribal community, but save as aforesaid a notification issued under the said clause shall not be varied by any subsequent notification.

Hence, statement 1 is incorrect and statement 2 is correct.

Article 366(25)-“Scheduled Tribes” means such tribes or tribal communities or parts of or groups within such tribes or tribal communities as are deemed under Article 342 to be Scheduled Tribes for the purposes of this Constitution.

The criteria generally adopted for the specification of a community as a scheduled tribe are not spelled out in the Constitution.

85. With reference to Union Budget, consider the following statements:

1. The Union Finance Minister on behalf of the Prime Minister lays the Annual Financial Statement before both the Houses of Parliament.

2. At the Union level, no demand for a grant can be made except on the recommendation of the President of India.

Which of the statements given above is/are correct?

(a) 1 only

(b) 2 only

(c) Both 1 and 2

(d) Neither 1 nor 2

85. Ans: b

Explanation: 

In all the parliamentary democracies, the budget, after having been prepared and passed by the executive government, is presented to the legislature, where it is discussed and finally voted. The responsibility of presenting the budget to the legislature is that of the executive government (and not the Prime Minister alone). Someone representing the executive government, usually the Finance Minister, must present the budget to the legislature. 

Centralised executive responsibility for budget preparation and submission will facilitate budget authorisation by the legislature and enable it to center attention on programme review and policy implementation. 

The executive right to present the budget to the legislature gives an opportunity to the executive government to formulate economic and financial policies of the country.

Hence, statement 1 is incorrect.

Article 112 in the Constitution of India:

Annual financial statement.—(1) The President shall in respect of every financial year cause to be laid before both the Houses of Parliament a statement of the estimated receipts and expenditure of the Government of India for that year, in this Part referred to as the “annual financial statement”.  

Article 113 (3) in the Constitution of India:

No demand for a grant shall be made except on the recommendation of the President.

Hence, statement 2 is correct.

86. Who of the following is the author of the books “The India Way” and “Why Bharat Matters”?

(a) Bhupender Yadav

(b) Nalin Mehta

(c) Shashi Tharoor

(d) Subrahmanyam Jaishankar

86. Ans: d

Explanation:

’The India Way’’ and ‘’Why Bharat Matters’’ is written by Subrahmanyam Jaishankar. 

Subrahmanyam Jaishankar is an Indian diplomat and politician serving as the Minister of External Affairs of the Government of India.

Hence, option d is correct.

87. Consider the following pairs:

CountryReason for being in the news
  1.ArgentinaWorst economic crisis
  2.SudanWar between the country’s regular army and paramilitary forces
  3.TurkeyRescinded its membership of NATO

How many of the pairs given above are correctly matched?

(a) Only one pair

(b) Only two pairs

(c) All three pairs

(d) None of the pairs

87. Ans: b

Explanation:

Argentina-With one of the world’s highest inflation rates and after more than a decade of economic stagnation and rising poverty, Argentina once again finds itself teetering on the brink of economic collapse.

Persistent fiscal deficits and chronic inflation are longstanding issues in Argentina.

Sudan -In April 2023, fighting between rival armed factions broke out in the Sudanese capital of Khartoum, raising fears of a return to full-scale civil war. The conflict is primarily a power struggle between the leaders of the Sudanese Armed Forces (SAF) and a powerful paramilitary group known as the Rapid Support Forces (RSF). The two groups, led by General Abdel Fattah al-Burhan and Mohamed Hamdan “Hemedti” Dagalo, respectively,

Turkey – Turkey’s parliament finally approved Sweden’s bid for NATO membership, and Turkish President Recep Tayyip Erdogan swiftly ratified the measure. Turkish leaders accused the Sweden  of harboring Kurdish terrorists. They demanded that Sweden tighten its anti-terrorism laws, extradite people accused of terrorist activities in Turkey, and resume arms sales to Turkey.  Turkey joined NATO IN 1952. At present, NATO has 32 member countries. Sweden joined in 2024. 

Both pairs 1 and 2 are correctly matched.

Hence, option b is correct.

88. Consider the following statements:

Statement-I : Sumed pipeline is a strategic route for Persian Gulf oil and natural gas shipments to Europe.

Statement-II : Sumed pipeline connects the Red Sea with the Mediterranean Sea.

Which one of the following is correct in respect of the above statements?

(a) Both Statement-I and Statement-II are correct and Statement-Il explains Statement-I

(b) Both Statement-I and Statement-II are correct, but Statement-II does not explain Statement-I 

(c) Statement-I is correct, but Statement-II is incorrect

(d) Statement-I is incorrect, but Statement-II is correct

88. Ans: a

Explanation:

The Suez Canal and the SUMED Pipeline are strategic routes for Persian Gulf crude oil, petroleum products, and liquefied natural gas (LNG) shipments to Europe and North America. Located in Egypt, the Suez Canal connects the Red Sea with the Mediterranean Sea, and it is a critical chokepoint because of the large volumes of energy commodities that flow through it. Chokepoints are narrow channels along widely used global sea routes that are critical to global energy security.

Hence, statement 1 is correct.

The 200-mile long SUMED Pipeline transports crude oil northbound through Egypt from the Red Sea to the Mediterranean Sea. Crude oil flows through two parallel pipelines that have a total maximum flow capacity of 2.8 million barrels per day. The SUMED Pipeline is the only alternative route to transport crude oil from the Red Sea to the Mediterranean Sea if ships cannot navigate through the Suez Canal. 

Hence, statement 2 is correct.

89. Consider the following statements:

1. The Red Sea receives very little precipitation in any form.

2. No water enters the Red Sea from rivers.

Which of the statements given above is/are correct?

(a) 1 only

(b) 2 only

(c) Both 1 and 2

(d) Neither 1 nor 2

89. Ans: c

Explanation:

The Red Sea region receives very little precipitation in any form, although prehistoric artifacts indicate that there were periods with greater amounts of rainfall. 

Hence, statement 1 is correct.

No water enters the Red Sea from rivers, and rainfall is scant; but the evaporation loss—in excess of 80 inches per year

Hence, statement 2 is correct.

The Red Sea, home to the second-longest coral reef system in the world, is a vital resource for the Kingdom of Saudi Arabia. The Red Sea provides 90% of the Kingdom’s potable water by desalinization (it is one of the world’s saltiest water masses), and it supports tourism, shipping, aquaculture, and fishing industries, which together contribute about 10%–20% of the country’s GDP.

90. According to the Environmental Protection Agency (EPA), which one of the following is the largest source of sulphur dioxide emissions?

(a) Locomotives using fossil fuels

(b) Ships using fossil fuels

(c) Extraction of metals from ores

(d) Power plants using fossil fuels

90. Ans: d

Explanation:

According to EPA, the largest source of SO2 in the atmosphere is the burning of fossil fuels by power plants and other industrial facilities. Smaller sources of SO2 emissions include: industrial processes such as extracting metal from ore; natural sources such as volcanoes; and locomotives, ships and other vehicles and heavy equipment that burn fuel with a high sulfur content.    

Hence, option d is correct.

91. Consider the following statements:

Statement-I: There is instability and worsening security situation in the Sahel region.

Statement-II: There have been military takeovers/coups d’etat in several countries of the Sahel region in the recent past.

Which one of the following is correct in respect of the above statements?

(a) Both Statement-I and Statement-II are correct and Statement-II explains Statement-I

(b) Both Statement-I and Statement-II are correct, but Statement-II does not explain Statement-I

(c) Statement-I is correct, but Statement-II is incorrect

(d) Statement-I is incorrect, but Statement-II is correct

91. Ans: a

Explanation:

Countries in the Sahel (comprising Burkina Faso, Chad, Mali, Mauritania, and Niger), along with neighbouring Central African Republic (CAR) are facing a medley of development challenges. Escalating insecurity, political instability including military takeovers, climate change, and overlapping economic shocks are making it even harder to achieve sustainable and inclusive development in one of the poorest parts of the world.

In 2022, conflict-related fatalities in these countries increased by over 40 percent. The deterioration of the security situation over the past decade has caused a humanitarian crisis, with more than 3 million people fleeing violence in Burkina Faso, Mali, and Niger, according to UNHCR.

Both Statement-I and Statement-II are correct and Statement-II explains Statement-I

Hence, option a is correct.

92. Consider the following statements:

Statement I: India does not import apples from the United States of America.

Statement II: In India, the law prohibits the import of Genetically Modified food without the approval of the competent authority.

Which one of the following is correct in respect of the above statements?

(a) Both Statement-I and Statement-II are correct and Statement-II explains Statement-I

(b) Both Statement-I and Statement-II are correct, but Statement-II does not explain Statement-I

(c) Statement-I is correct, but Statement-II is incorrect

(d) Statement-I is incorrect, but Statement-II is correct

92. Ans:  d

Explanation:

In June 2023 Indian Government removed an additional 20% duty was imposed on US apples in 2019. An additional 20% duty was imposed on US apples in 2019 in response to USA’s measure to increase tariffs on certain steel and aluminum products. There is no reduction on Most Favoured Nation (MFN) duty on apples which is still applicable on all imported apples including on USA at 50%.

The market share of the US apples was taken by other countries due to imposition of additional retaliatory duty on US apples. This is reflected as the import of apples from countries other than the US increased from US$ 160 million in FY 2018-19 to US$ 290 million in FY 2022-23. Turkey, Italy, Chile, Iran and New Zealand are other top exporters to India of apples which took the market share of the US. 

The import policy of Genetically Modified Food, Feed, Genetically Modified Organism (GMOs) and Living Modified Organisms (LMOs) has been notified by Directorate General of Foreign Trade under General Notes regarding Import Policy in ITC (HS) 2012, Schedule-1 (Import Policy). As per the policy, import of GM food requires prior approval of the Genetic Engineering Approval Committee (GEAC) constituted by the Ministry of Environment Forest and Climate Change.

Further, the Customs Act, 1962 is the basic statue which governs /regulates entry/exit of different categories of goods into or outside the country. Various allied laws and regulations also apply. It is the responsibility of the Customs to ensure that all the imported/ exported goods fulfill the prescribed legal and procedural requirements laid down under Customs act, 1962 and allied laws including payment of the duties leviable, if any.

Import of food products is regulated under the Food Safety and Standards Act (FSSAI), 2006. Indian Customs can clear food products including Genetically Modified(GM) food products only after necessary approval/No Objection Certificate(NOC) by FSSAI. FSSAI has informed that no genetically modified food has been cleared for import through the FSSAI locations.

Statement-I is incorrect, but Statement-II is correct

93. With reference to the Speaker of the Lok Sabha, consider the following statements:

While any resolution for the removal of the Speaker of the Lok Sabha Is under consideration

1. He/She shall not preside.

2. He/She shall not have the right to speak.

3. He/She shall not be entitled to vote on the resolution in the first instance.

Which of the statements given above is/are correct?

(a) 1 only

(b) 1 and 2 only

(c) 2 and 3 only

(d) 1, 2 and 3

93. Ans: a

Explanation:

The Speaker or the Deputy Speaker is removable from his office by a resolution of Lok Sabha passed by a majority of all the then members of the House. 

At least fourteen days’ notice has to be given of the intention to move such a resolution. A member wishing to give notice of a resolution for the removal of the Speaker or the Deputy Speaker is required to do so in writing to the Secretary-General 

At the sitting of Lok Sabha, while any resolution for the removal of the Speaker or the Deputy Speaker from his office is under consideration, the Speaker or the Deputy Speaker, as the case may be, cannot preside even though he is present in the House.

Hence, statement 1 is correct. 

The Speaker has the right to speak and otherwise to take part in the proceedings of Lok Sabha while any resolution for his removal from office is under consideration in the House and is entitled to vote only in the first instance on such resolution or any other matter during such proceedings but not in the case of an equality of vote.

Hence, both statements 2 and 3 are incorrect.

94. With reference to the Indian Parliament, consider the following statements:

1. A bill pending in the Lok Sabha lapses on its dissolution.

2. A bill passed by the Lok Sabha and pending in the Rajya Sabha lapses on the dissolution of the Lok Sabha.

3. A bill in regard to which the President of India notified his/her intention to summon the Houses to a joint sitting lapse on the dissolution of the Lok Sabha.

Which of the statements given above is/are correct?

(a) 1 only

(b) 1 and 2

(c) 2 and 3

(d) 3 only

94. Ans: b

Explanation:

The end of the life of Lok Sabha either by an order made by the President under article 85(2)(b) or on the expiration of the period of five years from the date appointed for its first meeting is termed as ‘dissolution of the House’

Article 107 lays down the effect of dissolution upon Bills before each House of Parliament in the event of dissolution of Lok Sabha. The present position of the effect of dissolution upon the pending Bills is that-

In Lok Sabha, all Bills pending at the time of dissolution, whether originating in the House or transmitted to it by Rajya Sabha, lapse. 

Hence, statement 1 is correct. 

In Rajya Sabha, Bills passed by Lok Sabha, but which have not been disposed of and are pending in Rajya Sabha on the date of dissolution, lapse. 

Hence, statement 2 is correct. 

Only the Bills originating in Rajya Sabha which have not been passed by Lok Sabha but are still pending before Rajya Sabha, do not lapse

If, however, in respect of a Bill upon which the Houses have disagreed and the President has notified his intention of summoning a Joint Sitting of the Houses for the consideration of the Bill prior to dissolution, that Bill does not lapse and may be passed at a Joint Sitting of both Houses, notwithstanding that dissolution has intervened since the President notified his intention to summon the Joint Sitting of the Houses  

Hence, statement 3 is incorrect. 

There is no express provision in the Constitution regarding the effect of dissolution on a Bill which has been passed by the two Houses of Parliament and sent to the President for assent. It has, however, been held that such a Bill does not lapse on dissolution of Lok Sabha. Further, if such a Bill is returned by the President for reconsideration, the successor House can reconsider it and if it is passed by the successor House (with or without amendments), it will be deemed to have been passed “again” 

95. With reference to the Parliament of India, consider the following statements:

1. Prorogation of a House by the President of India does not require the advice of the Council of Ministers.

2. Prorogation of a House is generally done after the House is adjourned sine die but there is no bar to the President of India prorogating the House which is in session.

3. Dissolution of the Lok Sabha is done by the President of India who, save in exceptional circumstances, does so on the advice of the Council of Ministers.

Which of the statements given above is/are correct?

(a) 1 only

(b) 1 and 2

(c) 2 and 3

(d) 3 only

95. Ans: c

Explanation:

Termination of a session of the House by an order made by the President under article 85(2) is called ‘prorogation’.

The President, in exercising the power to prorogue the House, acts on the advice of the Prime Minister. The Prime Minister may consult the Cabinet before the advice is submitted to the President.  

Prorogation of the House may take place any time, even while the House is sitting. Usually, however, prorogation follows the adjournment of the sitting of the House sine die 

 After the adjournment of Lok Sabha sine die, the Minister of Parliamentary Affairs (or the Leader of the House, as the case may be) sends a communication to the Secretary-General conveying the intention of the Prime Minister or the Cabinet to prorogue the House. The proposal of the Prime Minister, after being agreed to by the Speaker, is submitted to the President

Hence, statement 1 is incorrect and statement 2 is correct.

The power to dissolve Lok Sabha is vested in the President and he exercises this power on the advice of the Prime Minister. The Prime Minister may consult the Cabinet or inform it of his intention to make a recommendation to the President.  

Article 53 of the Constitution provides that the executive power of the Union vests in the President. Under Article 74(1) all the functions of the President have to be discharged by him with the aid and advice of the Council of Ministers headed by the Prime Minister. Therefore, it is contended that the exercise of the power of dissolution is an executive function of the President’s office and as such has to be performed with the aid and advice of the Council of Ministers. 

However every power conferred by the Constitution on any authority, must be exercised in accordance with the Constitution and in accordance with valid laws enacted under it, or continued by it. Thus if the President feels that the advice of the Council of Ministers defeats the underlying principles of democratic government, he must act with discretion and wisdom (exceptional circumstance).

Thus in India as in England, dissolution may be rejected if the President opines that 

  • The existing Parliament is still vital, viable and capable of doing its job.
  • A general election would be detrimental to the national economy,” and 
  • The President could find another Prime Minister who would carry on his government for a reasonable period with a working majority in the House.

Hence, statement 3 is correct. 

96. Consider the following statements:

Statement-I: The European Parliament approved The Net-Zero Industry Act recently.

Statement-II: The European Union intends to achieve carbon neutrality by 2040 and therefore aims to develop all of its own clean technology by that time.

Which one of the following is correct in respect of the above statements?

(a) Both Statement-I and Statement-II are correct and Statement-II explains Statement-I

(b) Both Statement-I and Statement-II are correct, but Statement-II does not explain Statement-I               ·

(c) Statement-I is correct, but Statement-II is incorrect

(d) Statement-I is incorrect, but Statement-II is correct

96. Ans: c 

Explanation:

In March 2023 the European Commission introduced the Net Zero Industry Act, which proposes to establish a framework of measures to strengthen Europe’s net-zero technology products manufacturing ecosystem. The overall goal is to put Europe on a path to domestically manufacture at least 40% of its clean energy technology needs by 2030. 

Hence, statement 1 is correct & statement 2 is incorrect.

97. Consider the following statements:

Statement-I: Recently, Venezuela has achieved a rapid recovery from its economic crisis and succeeded in preventing its people from fleeing/emigrating to other countries

Statement II: Venezuela has the world’s largest oil reserves.

Which one of the following is correct in respect of the above statements?

(a) Both Statement-I and Statement-II are correct and Statement-II explains Statement-I

(b) Both Statement-I and Statement-II are correct, but Statement-II does not explain Statement-I

(c) Statement-I is correct, but Statement-II is incorrect

(d) Statement-I is incorrect, but Statement-II is correct

97. Ans: d

Explanation:

Venezuela has been in a severe socio-political and economic crisis for several years.

With an exodus of more than 7.72 million people since 2014, the refugee crisis in Venezuela is the largest displacement crisis in Latin America and one of the largest in the world.

Several root causes generate this unprecedented flow of migrants and refugees, including a deep economic crisis, repression and democratic breakdown in Venezuela.  

Hence, statement 1 is incorrect 

Venezuela has the largest oil reserves in the world, with 24.8% of the world’s proven oil reserves. Venezuela’s economy is heavily dependent on oil revenues, which make up 96% of its exports and 11.6% of its GDP.

United States has imposed sanctions on Venezuelan crude oil.

Hence, statement 2 is correct.

98. With reference to the Digital India Land Records Modernisation Programme, consider the following statements:

1. To implement the scheme, the Central Government provides 100% funding.

2. Under the Scheme, Cadastral Maps are digitised.

3. An initiative has been undertaken to transliterate the Records of Rights from the local language to any of the languages recognized by the Constitution of India.

Which of the statements given above are correct?

(a) 1 and 2 only

(b) 2 and 3 only                

(c) 1 and 3 only

(d) 1, 2 and 3

98. Ans: d                                                             

Explanation:

Digital India Land Records Modernization Programme

Digital India Land Records Modernization Programme (erstwhile National Land Record Modernization Programme-) was revamped and converted as a Central Sector Scheme with effect from 1st April 2016 with 100% funding by the Centre. 

The objective of DILRMP is to develop a modern, comprehensive and transparent land record management system with the aim to develop an Integrated Land Information Management System that will inter alia

(i) improve the real-time information on land; 

(ii) optimize use of land resources; 

(iii) benefit both landowners & prospectors; 

(iv) assist in policy & planning; 

(v) reduce land disputes; 

(vi) check fraudulent/benami transactions 

(vii) obviate need of physical visits to Revenue/Registration offices 

(viii) enable sharing of information with various organisations/agencies.

Hence, statement 1 is correct.

The government has approved the extension of DILRMP for a period of five years i.e. 2021-22 to 2025-26.

Two new components viz Consent –based integration of Aadhaar number with the land record database and Computerization of Revenue Courts and their integration with land records have also been now added as part of DILRMP in addition to components of the programme viz. 

(i) Setting up of Modern Record Room (Teshil) 

(ii) Survey/re-survey (iii) Data entry/re-entry

(iv) Digitization of cadastral Maps/FMBs/Tippans 

(v) State Level Data Centre 

(vi) Computerization of Registration process 

(vii) DILRMP Cell

(vii) PMU 

(xi) Evaluation Studies, IEC and Training 

(x) Core GIS/Software Applications.

Hence, statement 2 is correct.

Some innovative initiatives under DILRMP:

a) Unique Land Parcel Identification Number (ULPIN) or Bhu-Aadhar

b) National Generic Document Registration System (NGDRS) or E-Registration

c) Linkage of e-Court with Land Records / Registration Database

d) Transliteration of Land Records in all languages of Schedule VIII in all States/UTs:

Currently, the Records of Rights in States and Union Territories are maintained in local languages. In order to address the problem of linguistic barriers in land governance, the Government has undertaken an initiative to transliterate the Records of Rights available in the local language to any of the 22 Schedule VIII languages of the Constitution. 

Hence, statement 3 is correct.

99. With reference to the ‘Pradhan Mantri Surakshit Matritva Abhiyan’, consider the following statements:

1. This scheme guarantees a minimum package of antenatal care services to women in their second and third trimesters of pregnancy and six months post-delivery health care service in any government health facility.

2. Under this scheme, private sector healthcare providers of certain specialities can volunteer to provide services at nearby government health facilities.

Which of the statements given above is/are correct?

(a) 1 only

(b) 2 only

(c) Both 1 and 2

(d) Neither 1 nor 2

99. Ans: b

Explanation:

Pradhan Mantri Surakshit Matritva Abhiyan

The Pradhan Mantri Surakshit Matritva Abhiyan has been launched by the Ministry of Health & Family Welfare (MoHFW), Government of India. The program aims to provide assured, comprehensive and quality antenatal care, free of cost, universally to all pregnant women on the 9th of every month.

Hence, statement 1 is incorrect.

PMSMA guarantees a minimum package of antenatal care services to women in their 2nd / 3rd trimesters of pregnancy at designated government health facilities

The programme follows a systematic approach to engagement with the private sector, which includes motivating private practitioners to volunteer for the campaign, developing strategies for generating awareness and appealing to the private sector to participate in the Abhiyan at government health facilities.

Hence, statement 2 is correct.

100. With reference to the Pradhan Mantri Shram Yogi Maan-dhan (PM-SYM) Yojana, consider the following statements:

1. The entry age group for enrolment in the scheme is 21 to 40 years.

2. Age specific contribution shall be made by the beneficiary.

3. Each subscriber under the scheme shall receive a minimum pension of ₹ 3,000 per month after attaining the age of 60 years.

4. Family pension is applicable to the spouse and unmarried daughters.

Which of the statements given above is/are correct?

(a) 1, 3 and 4

(b) 2 and 3 only

(c) 2 only

(d) 1, 2 and 4 

100. Ans: b

Explanation:

PRADHAN MANTRI SHRAM YOGI MAAN-DHAN [PM-SYM] YOJANA

The Government of India has introduced a pension scheme for unorganised workers, Pradhan Mantri Shram Yogi Maan-dhan (PM-SYM), to ensure their old age protection for the unorganised workers whose monthly income is Rs 15,000/ per month or less and belong to the entry age group of 18-40 years. They should not be covered under the New Pension Scheme (NPS), the Employees State Insurance Corporation (ESIC) scheme or the Employees‟ Provident Fund Organisation (EPFO). Further, he/she should not be an income taxpayer. 

Hence, statement 1 is incorrect.

Features of PM-SYM: It is a voluntary and contributory pension scheme, under which the subscriber would receive the following benefits:

(i) Minimum Assured Pension: Each subscriber under the PM-SYM, shall receive minimum assured pension of Rs 3000/- per month after attaining the age of 60 years. 

Hence, statement 3 is correct.

(ii) Family Pension: During the receipt of pension, if the subscriber dies, the spouse of the beneficiary shall be entitled to receive 50% of the pension received by the beneficiary as family pension. Family pension is applicable only to spouse. 

Hence, statement 4 is incorrect.

(iii) If a beneficiary has given regular contribution and died due to any cause (before age of 60 years), his/her spouse will be entitled to join and continue the scheme subsequently by payment of regular contribution or exit the scheme as per provisions of exit and withdrawal.

Contribution by the Subscriber: The subscriber’s contributions to PM-SYM shall be made through ‘auto-debit’ facility from his/ her savings bank account/ Jan- Dhan account. The subscriber is required to contribute the prescribed contribution amount from the age of joining PM-SYM till the age of 60 years.

Matching contribution by the Central Government: PM-SYM is a voluntary and contributory pension scheme on a 50:50 basis where prescribed age-specific contribution shall be made by the beneficiary and the matching contribution by the Central Government 

Hence, statement 2 is correct.

Fund Management: PM-SYM will be a Central Sector Scheme administered by the Ministry of Labour and Employment and implemented through Life Insurance Corporation of India and CSC e-Governance Services India Limited (CSC-SPV).

Pension Pay out: Once the beneficiary joins the scheme at the entry age of 18-40 years, the beneficiary has to contribute till 60 years of age. On attaining the age of 60 years, the subscriber will get the assured monthly pension of Rs.3000/- with benefit of family pension, as the case may be.

Leave A Comment

Your email address will not be published. Required fields are marked *